Combo with "Mental Health week 1" and 10 others

¡Supera tus tareas y exámenes ahora con Quizwiz!

A geriatric client is confused and wandering in and out of every door. Which scenario reflects the least restrictive alternative for this client? A. The client is placed in seclusion. B. The client is placed in a geriatric chair with tray. C. The client is placed in soft Posey restraints. D. The client is monitored by an ankle bracelet.

ANS: D The least restrictive alternative for this client would be monitoring by an ankle bracelet. The client does not pose a direct dangerous threat to self or others, so neither physical restraints nor seclusion would be justified.

Which situation reflects the ethical principle of veracity? A. A nurse provides a client with outpatient resources to benefit recovery. B. A nurse refuses to give information to a physician who is not responsible for the client's care. C. A nurse tricks a client into seclusion by asking the client to carry linen to the seclusion room. D. A nurse treats all of the clients equally regardless of illness severity.

ANS: C The nurse who tricks a client into seclusion has violated the ethical principle of veracity. The principle of veracity refers to one's duty to always be truthful and not intentionally deceive or mislead clients.

A nurse holds the hand of a client who is withdrawing from alcohol. What is the nurse's rationale for this intervention? A. To assess for emotional strength B. To assess for Wernicke-Korsakoff syndrome C. To assess for tachycardia D. To assess for fine tremors

ANS: D The nurse is most likely assessing the client for fine tremors secondary to alcohol withdrawal. Withdrawal from alcohol can also cause headache, insomnia, transient hallucinations, depression, irritability, anxiety, elevated blood pressure, sweating, tachycardia, malaise, coarse tremors, and seizure activity.

Which therapeutic communication technique is being used in this nurse-client interaction? Client: "When I get angry, I get into a fistfight with my wife or I take it out on the kids." Nurse: "I notice that you are smiling as you talk about this physical violence." A. Encouraging comparison B. Exploring C. Formulating a plan of action D. Making observations

ANS: D The nurse is using the therapeutic communication technique of making observations when noting that the client smiles when talking about physical violence. The technique of making observations encourages the client to compare personal perceptions with those of the nurse.

Which medication orders should a nurse anticipate for a client who has a history of complicated withdrawal from benzodiazepines? A. Haloperidol (Haldol) and fluoxetine (Prozac) B. Carbamazepine (Tegretol) and donepezil (Aricept) C. Disulfiram (Antabuse) and lorazepan (Ativan) D. Chlordiazepoxide (Librium) and phenytoin (Dilantin)

ANS: D The nurse should anticipate that a physician would order chlordiazepoxide (Librium) and phenytoin (Dilantin) for a client who has a history of complicated withdrawal from benzodiazepines. It is common for long-lasting benzodiazepines to be prescribed for substitution therapy. Phenytoin (Dilantin) is an anticonvulsant that would be indicated for a client who has experienced a complicated withdrawal. Complicated withdrawals may progress to seizure activity.

On the first day of a client's alcohol detoxification, which nursing intervention should take priority? A. Strongly encourage the client to attend 90 Alcoholics Anonymous meetings in 90 days. B. Educate the client about the biopsychosocial consequences of alcohol abuse. C. Administer ordered chlordiazepoxide (Librium) in a dosage according to protocol. D. Administer vitamin B1 to prevent Wernicke-Korsakoff syndrome.

ANS: C The priority nursing intervention for this client should be to administer ordered chlordiazepoxide (Librium) in a dosage according to protocol. Chlordiazepoxide (Librium) is a benzodiazepine and is often used for substitution therapy in alcohol withdrawal. Substitution therapy may be required to reduce life-threatening effects of the rebound stimulation of the central nervous system that occurs during withdrawal.

Which term should a nurse use to describe the administration of a central nervous system (CNS) depressant during alcohol withdrawal? A. Antagonist therapy B. Deterrent therapy C. Codependency therapy D. Substitution therapy

ANS: D A CNS depressant such as Ativan is used during alcohol withdrawal as substitution therapy to prevent life-threatening symptoms that occur because of the rebound reaction of the central nervous system.

Which client statement demonstrates positive progress toward recovery from substance abuse? A. "I have completed detox and therefore am in control of my drug use." B. "I will faithfully attend Narcotic Anonymous (NA) when I can't control my carvings." C. "As a church deacon, my focus will now be on spiritual renewal." D. "Taking those pills got out of control. It cost me my job, marriage, and children."

ANS: D A client who takes responsibility for the consequences of substance abuse/dependence is making positive progress toward recovery. This client would most likely be in the working phase of the counseling process in which acceptance of the fact that substance abuse causes problems occurs.

Which statement should a nurse identify as the most accurate description of the DSM-IV-TR's definition of a mental disorder? A. A mental disorder is said to exist when an individual's behavior deviates from the group norm. B. A mental disorder results when an individual is unable to cope with the stressors of everyday life. C. A mental disorder exists when subjective distress causes the individual to fail at many tasks. D. A mental disorder is a clinically significant behavioral or psychological pattern that is associated with present distress or disability.

ANS: D The nurse should identify that a mental disorder indicates a pattern of maladaptive clinically significant behavioral or psychological problems. The DSM-IV-TR also specifies that the behavior or pattern is not an expectable and culturally sanctioned response to a particular event.

On which task should a nurse place priority during the working phase of relationship development? A. Establishing a contract for intervention B. Examining feelings about working with a particular client C. Establishing a plan for continuing aftercare D. Promoting the client's insight and perception of reality

ANS: D The nurse should place priority on promoting the client's insight and perception of reality during the working phase of relationship development. Establishing a contract for intervention would occur in the orientation phase. Examining feelings about working with a client should occur in the preinteraction phase. Establishing a plan for aftercare would occur in the termination phase.

Which is the best nursing action when a client demonstrates transference toward a nurse? A. Promoting safety and immediately terminating the relationship with the client B. Encouraging the client to ignore these thoughts and feelings C. Immediately reassigning the client to another staff member D. Helping the client to clarify the meaning of the current nurse-client relationship

ANS: D The nurse should respond to a client's transference by clarifying the meaning of the nurse-client relationship based on the current situation. Transference occurs when the client unconsciously displaces feelings toward the nurse about a person from the past. The nurse should assist the client in separating the past from the present.

What are the three sources of law?

- common law - statutory law - administrative law

Axis __ is clinical disorders such as schizophrenia, major depression, and bipolar disorder

1

The M'Naghten rule is successful in about what percentage of cases?

1%

d Prescriptive privileges are granted to masters-prepared nurse practitioners who have taken special courses on prescribing medication. The nurse prepared at the basic level is permitted to perform mental health assessments, establish relationships, and provide individualized care planning.

1. A new staff nurse completes orientation to a psychiatric unit. This nurse may expect an advanced practice nurse to perform which additional interventions? a. Conduct mental health assessments b. Establish therapeutic relationships c. Individualize nursing care plans d. Prescribe psychotropic medication

Which should the nurse recognize as an example of the defense mechanism of repression? A. A student aware of the need to study for tomorrow's test goes to a movie instead. B. A woman whose son was killed in Iraq does not believe the military report. C. A man who is unhappily married goes to school to become a marriage counselor. D. A woman was raped when she was 12 and no longer remembers the incident.

ANS: D Repression is the involuntary blocking of unpleasant feelings and experiences from one's awareness.

b

1. J has been admitted to the psychiatric hospital for assessment and evaluation. What behavior might indicate that J has a mental illness? a. She is able to see the difference between the "as if" and the "for real." b. She describes her mood as consistently sad, discouraged, down in the dumps, hopeless. c. She responds to the rules, routines, and customs of any group to which she belongs. d. Within the limits set by her abilities, she can perform tasks she attempts.

When is self-disclosure by the nurse appropriate in a therapeutic nurse-client relationship? A. When it is judged that the information may benefit the nurse and client B. When the nurse has a duty to warn C. When the nurse feels emotionally indebted toward the client D. When it is judged that the information may benefit the client

ANS: D Self-disclosure on the part of the nurse may be appropriate when it is judged that the information may therapeutically benefit the client. It should never be undertaken for the purpose of meeting the nurse's needs.

What percentage of the homeless have a severe mental illness?

20-25%

b

3. Strength of the multiaxial DSM-IV is that it: a. includes nursing as well as medical diagnoses b. assessments of several aspects of functioning are made c. it employs the framework of a specific theory d. plans for treatment and evaluation are included

d The crisis of autonomy versus shame and doubt relates to the developmental task of gaining control of self and environment, as exemplified by toilet training. This psychosocial crisis occurs during the period of early childhood. Trust versus mistrust is the crisis of the infant. Initiative versus guilt is the crisis of the preschool and early-school-aged child. Industry versus inferiority is the crisis of the 6- to 12-year-old child.

A 26-month-old displays negative behavior, refuses toilet training, and often says, "No!" Which psychosocial crisis is evident? a. Trust versus mistrust b. Initiative versus guilt c. Industry versus inferiority d. Autonomy versus shame and doubt

d Although the nurse is accountable to the health care provider, the agency, the patient, and the profession, the duty of care is owed to the patient.

A newly admitted acutely psychotic patient is a private patient of the chief of staff and a private-pay patient. To whom does the psychiatric nurse assigned to the patient owe the duty of care? a. Health care provider b. Hospital c. Profession d. Patient

Set limits on destructive behavior. Direct a patient to go to a quiet place. Sit with a withdrawn, isolated patient. Distract a patient who is hallucinating.

A patient tells the nurse, "I want to have sex with you." Which nursing responses are appropriate? "Sex is not part of our relationship." "We are here to work on your problems."

Which statement should a nurse identify as correct regarding a client's right to refuse treatment? A. Clients can refuse pharmacological but not psychological treatment. B. Clients can refuse any treatment at any time. C. Clients can refuse only electroconvulsive therapy (ECT). D. Professionals can override treatment refusal if the client is actively suicidal or homicidal.

ANS: D The nurse should understand that health-care professionals can override treatment refusal when a client is actively suicidal or homicidal. A suicidal or homicidal client who refuses treatment may be a danger to self or others. This situation should be treated as an emergency, and treatment may be performed without informed consent.

What is the legal significance of a nurse's action when the nurse threatens a demanding client with restraints? A. The nurse can be charged with assault. B. The nurse can be charged with negligence. C. The nurse can be charged with malpractice. D. The nurse can be charged with beneficence.

ANS: A Assault is an act that results in a person's genuine fear and apprehension that he or she will be touched without consent.

Which client should the nurse anticipate to be most receptive to psychiatric treatment? A. A Jewish, female journalist B. A Baptist, homeless male C. A Catholic, black male D. A Protestant, Swedish business executive

ANS: A The nurse should anticipate that the client of Jewish culture would place a high importance on preventative health care and would consider mental health as equally important as physical health. Women are also more likely to seek treatment for mental health problems than men.

A new psychiatric nurse states, "This client's use of defense mechanisms should be eliminated." Which is a correct evaluation of this nurse's statement? A. Defense mechanisms can be appropriate responses to stress and need not be eliminated. B. Defense mechanisms are a maladaptive attempt of the ego to manage anxiety and should always be eliminated. C. Defense mechanisms, used by individuals with weak ego integrity, should be discouraged and not eliminated. D. Defense mechanisms cause disintegration of the ego and should be fostered and encouraged.

ANS: A The nurse should determine that defense mechanisms can be appropriate during times of stress. The client with no defense mechanisms may have a lower tolerance for stress thus leading to anxiety disorders. Defense mechanisms should be confronted when they impede the client from developing healthy coping skills.

A lonely, depressed divorcée has been self-medicating with cocaine for the past year. Which term should a nurse use to best describe this individual's situation? A. The individual is experiencing psychological dependency. B. The individual is experiencing physical dependency. C. The individual is experiencing substance dependency. D. The individual is experiencing social dependency.

ANS: A The nurse should use the term "psychological dependency" to best describe this client's situation. A client is considered to be psychologically dependent on a substance when there is an overwhelming desire to use a substance in order to produce pleasure or avoid discomfort.

A mother notified that her child was killed in a tragic car accident states, "I can't bear to go on with my life." Which nursing statement conveys empathy? A. "This situation is very sad, but time is a great healer." B. "You are sad, but you must be strong for your other children." C. "Once you cry it all out, things will seem so much better." D. "It must be horrible to lose a child; I'll stay with you until your husband arrives."

ANS: D The nurse's response, "It must be horrible to lose a child; I'll stay with you until your husband arrives." conveys empathy to the client. Empathy is the ability to see the situation from the client's point of view. Empathy is considered to be one of the most important characteristics of the therapeutic relationship.

A nurse states to a client, "Things will look better tomorrow after a good night's sleep." This is an example of which communication technique? A. The therapeutic technique of "giving advice" B. The therapeutic technique of "defending" C. The nontherapeutic technique of "presenting reality" D. The nontherapeutic technique of "giving false reassurance"

ANS: D The nurse's statement, "Things will look better tomorrow after a good night's sleep." is an example of the nontherapeutic technique of giving false reassurance. Giving false reassurance indicates to the client that there is no cause for anxiety, thereby devaluing the client's feelings.

During a group session, which client statement demonstrates that the group has progressed to the middle, or working, phase of group development? A. "It's hard for me to tell my story when I'm not sure about the reactions of others." B. "I think Joe's Antabuse suggestion is a good one and might work for me." C. "My situation is very complex, and I need professional, not peer, advice." D. "I am really upset that you expect me to solve my own problems."

ANS: B The nurse should determine that group members have progressed to the working phase of group development when members begin to look to each other instead of to the leader for guidance. Group members in the working phase begin to accept criticism from each other and then use it constructively to foster change.

The nurse should utilize which group function to help an extremely withdrawn, paranoid client increase feelings of security? A. Socialization B. Support C. Empowerment D. Governance

ANS: B The nurse should identify that the group function of support would help an extremely withdrawn, paranoid client increase feelings of security. Support assists group members in gaining a feeling of security from group involvement.

A brother calls to speak to his sister who has been admitted to the psychiatric unit. The nurse connects him to the community phone and the sister is summoned. Later the nurse realizes that the brother was not on the client's approved call list. What law has the nurse broken? A. The National Alliance for the Mentally Ill Act B. The Tarasoff Ruling C. The Health Insurance Portability and Accountability Act D. The Good Samaritan Law

ANS: C The nurse has violated the Health Insurance Portability and Accountability Act (HIPAA) by revealing that the client had been admitted to the psychiatric unit. The nurse should not have provided any information without proper consent from the client.

A client's younger daughter is ignoring curfew. The client states, "I'm afraid she will get pregnant." The nurse responds, "Hang in there. Don't you think she has a lot to learn about life?" This is an example of which communication block? A. Requesting an explanation B. Belittling the client C. Making stereotyped comments D. Probing

ANS: C This is an example of the nontherapeutic communication block of making stereotyped comments. Clichés and trite expressions are meaningless in a therapeutic nurse-client relationship.

Which should the nurse recognize as a DSM-IV-TR Axis II disorder? A. Obesity B. Major depressive disorder C. Hypertension D. Borderline personality disorder

ANS: D Axis II records personality disorders and mental retardation. These disorders begin in childhood or adolescence and persist in a stable form into adult life.

A nurse should recognize that the diagnostic and statistical manual of mental disorders (DSM-IV-TR) is based on which model of health-care delivery? A. The holistic nursing model B. The eclectic model C. Freud's psychoanalytic model D. The medical model

ANS: D Psychiatric diagnoses are presented by the American Psychiatric Association (APA) on a multiaxial evaluation system. Psychiatrists and nurse practitioners can assign client data to the various axes. This is based on a medical model of health-care delivery.

In the role of milieu manager, which activity should the nurse prioritize? A. Setting the schedule for the daily unit activities B. Evaluating clients for medication effectiveness C. Conducting therapeutic group sessions D. Searching newly admitted clients for hazardous objects

ANS: D The milieu manager should search newly admitted clients for hazardous objects. Safety of the client and others is the priority. Nurses are responsible for ensuring that the client's safety and physiological needs are met within the milieu.

Which client statement indicates a knowledge deficit related to substance abuse? A. "Although it's legal, alcohol is one of the most widely abused drugs in our society." B. "Tolerance to heroin develops quickly." C. "Flashbacks from LSD use may reoccur spontaneously." D. "Marijuana is like smoking cigarettes. Everyone does it. It's essentially harmless."

ANS: D The nurse should determine that the client has a knowledge deficit related to substance abuse when the client compares marijuana to smoking cigarettes and claims it to be harmless. Cannabis is the second most widely abused drug in the United States.

Who developed the theory of motivation, which established the usefulness of talking, explained the importance of dreams, discussed sex openly and introduced terms that have become part of our language today?

Freud

What was the first psychiatric nurse?

Linda Richards

c

S is a newly admitted acutely psychotic client. She is a private client of the chief of staff and a private-pay client. To whom does the psychiatric nurse caring for S owe the duty of care? a. Physician b. Health care agency c. Client d. Profession

b The correct response describes a mood alteration, which further reflects mental illness. The distracters describe mentally healthy behaviors.

Which finding best indicates that a patient has a mental illness? The patient: a. responds to rules, routines, and customs of a group. b. reports mood is consistently sad, discouraged, and hopeless. c. performs tasks attempted within the limits set by own abilities. d. is able to see the difference between the "as if" and the "for real."

a

Which remark by the nurse would be an appropriate way to begin a therapeutic 1:1 session? a. "How shall we start today?" b. "Shall we talk about losing your privileges yesterday?" c. "What happened when your husband came to visit yesterday?" d. Tell me what led up to your hospitalization?

_____ law is public law issued by administrative agencies authorized by state statutes to administers the enacted laws of federal and state governments

administrative law

What benchmark was the decade of the brain?

benchmark 5

historically, who was responsible for developing asylums in the united states?

dorothea dix

Mental illness is not equivalent to ____

incompetence

What is battery?

intentional touching of another in a manner that violates physical security of another

milieu management

safety, structure, norms, setting limits, balance, independence versus dependence, environmental modification

What is M'Naghten rule?

states that individuals who do not understand the nature and implications of murderous actions because of insanity cannot be held legally accountable for murder

Which period began with the discovery of the first drugs to treat mental illness?

the period of psychotropic drugs

What question is always asked during court cases about negligence?

"Did the nurse meet the standard of care?"

A new nurse says, "I have more important things to do than play games with patients. These activities are not a worthwhile use of my time." Select the nurse manager's best response.

"Informal activities help patients develop social skills and take risks."

What two federal actions supported deinstitutionalization?

- Community Mental Health Center Act of 1963 - federal legislation that provided mentally disabled persons with an income while living in the community

What are the four elements that must be present for a plaintiff to recover damages caused by negligence?

- duty to care - reasonable care (standard of care) - breach of duty - proximate cause or causation

What were the only treatments for mental illness prior to thorazine?

- electroshock therapy - institutionalization - lobotomy

What are the three categories of involuntary treatment?

- emergency care - short term observation and treatment - long term commitment

What are some places that individuals with severe mental illness live?

- nursing homes - prisons/jails - state hospitals - homeless - home with families, group or board-and-care homes, or on their own

What did Wyatt vs. Stickney mandate that the state do?

- stop using patients for hospital labor needs - ensure a humane environment - develop and maintain minimal staffing standards - establish institutional human rights committees - provide the least restrictive environment for each patient

Axis __ is personality or development disorders such as paranoid and borderline personality disorders, and mental retardation

2

c

2. An outcome for S is that she will demonstrate mentally healthy behavior. Which behavior indicates the outcome is being met? a. She is aggressive in meeting her needs without considering the rights of others. b. She behaves without considering the consequences of her actions. c. She sees herself as approaching her ideals, and as capable of meeting demands. d. She passively allows others to assume responsibility for major areas of her life.

c Systematic desensitization is a type of therapy aimed at extinguishing a specific behavior, such as the fear of flying. Psychoanalysis and short-term dynamic therapy seek to uncover conflicts. Milieu therapy involves environmental factors.

A college student received an invitation to attend the wedding of a close friend who lives across the country. The student is afraid of flying. What type of therapy would the nurse suggest? a. Psychoanalysis b. Milieu therapy c. Systematic desensitization d. Short-term dynamic therapy

Transference involves a patient's emotional reaction to the nurse that is actually based on an earlier relationship or experience

A nurse considers interventions for a diabetic patient who needs to change eating habits and lose weight. The nurse will base strategies on which principle?

d The individual will be living up to the ego ideal, which will result in positive feelings about self. The other options are incorrect because each represents a negative feeling.

A nurse supports a parent for praising a child behaving in a helpful way. When this child behaves with politeness and helpfulness in adulthood, which feeling will most likely result? a. Guilt b. Anxiety c. Humility d. Self-esteem

c The DSM-IV-TR details the diagnostic criteria for psychiatric clinical conditions. The other references are good resources but do not define the diagnostic criteria.

A nurse wants to find a description of diagnostic criteria for anxiety disorders. Which resource would have the most complete information? a. The ICD-10 b. Nursing Outcomes Classification c. Diagnostic and Statistical Manual of Mental Disorders d. The ANA Psychiatric-Mental Health Nursing Scope and Standards of Practice

d Interpersonal psychotherapy returned the patient to his former level of functioning by helping him come to terms with the loss of friends and guilt over being a survivor. Milieu therapy refers to environmental therapy. Psychoanalysis would call for a long period of exploration of unconscious material. Behavior modification would focus on changing a behavior rather than helping the patient understand what is going on in his life.

A person says, "I was the only survivor in a small plane crash. Three business associates died. I got depressed and saw a counselor twice a week for 4 weeks. We talked about my feelings related to being a survivor and I'm OK now." Which type of therapy was used? a. Milieu therapy b. Psychoanalysis c. Behavior modification d. Interpersonal psychotherapy

The nurse caring for a hyperactive patient should be particularly concerned about assessing: physical safety.

A psychiatric aide asks, "Can you give me some examples of how we provide structure for patients?" The nurse should offer which suggestions? Select all that apply

Patient-initiated change is more successful than imposed change.

A psychotic patient tells the nurse, "Get away from me or I'll hit you. You're sucking the thoughts out of my head." The nurse should: increase the distance between self and patient.

Without authorization, a nurse administers an extra dose of narcotic tranquilizer to an agitated client. The nurse's coworker observes this action but does nothing for fear of repercussion. What is the ethical interpretation of the coworker's lack of involvement? A. Taking no action is still considered an unethical action by the coworker. B. Taking no action releases the coworker from ethical responsibility. C. Taking no action is advised when potential adverse consequences are foreseen. D. Taking no action is acceptable because the coworker is only a bystander.

ANS: A The coworker's lack of involvement can be interpreted as an unethical action. The coworker is experiencing an ethical dilemma in which a decision needs to be made between two unfavorable alternatives. The coworker has a responsibility to report any observed unethical actions.

After assertiveness training, a formerly passive client appropriately confronts a peer in group therapy. The group leader states, "I'm so proud of you for being assertive. You are so good!" Which communication technique has the leader employed? A. The nontherapeutic technique of giving approval B. The nontherapeutic technique of interpreting C. The therapeutic technique of presenting reality D. The therapeutic technique of making observations

ANS: A The group leader has employed the nontherapeutic technique of giving approval. Giving approval implies that the nurse has the right to pass judgment on whether the client's ideas or behaviors are "good" or "bad." This creates a conditional acceptance of the client.

A psychiatric nurse working on an inpatient unit receives a call asking if an individual has been a client in the facility. Which nursing response reflects appropriate legal and ethical obligations? A. Refusing to give any information to the caller, citing rules of confidentiality B. Refusing to give any information to the caller by hanging up C. Affirming that the person has been seen at the facility but providing no further information D. Suggesting that the caller speak to the client's therapist

ANS: A The most appropriate action by the nurse is to refuse to give any information to the caller. Admission to the facility would be considered protected health information (PHI) and should not be disclosed by the nurse without prior client consent.

A client diagnosed with chronic alcohol dependency is being discharged from an inpatient treatment facility after detoxification. Which client outcome related to Alcoholics Anonymous (AA) would be most appropriate for a nurse to discuss with the client during discharge teaching? A. After discharge, the client will immediately attend 90 AA meetings in 90 days. B. After discharge, the client will rely on an AA sponsor to help control alcohol cravings. C. After discharge, the client will incorporate family in AA attendance. D. After discharge, the client will seek appropriate deterrent medications through AA.

ANS: A The most appropriate client outcome for the nurse to discuss during discharge teaching is attending 90 AA meetings in 90 days after discharge. AA is a major self-help organization for the treatment of alcoholism. It accepts alcoholism as an illness and promotes total abstinence as the only cure.

What is the most essential task for a nurse to accomplish prior to forming a therapeutic relationship with a client? A. To clarify personal attitudes, values, and beliefs B. To obtain thorough assessment data C. To determine the client's length of stay D. To establish personal goals for the interaction

ANS: A The most essential task for a nurse to accomplish prior to forming a therapeutic relationship with a client is to clarify personal attitudes, values, and beliefs. Understanding one's own attitudes, values, and beliefs is called self-awareness.

Which therapeutic communication technique is being used in this nurse-client interaction? Client: "My father spanked me often." Nurse: "Your father was a harsh disciplinarian." A. Restatement B. Offering general leads C. Focusing D. Accepting

ANS: A The nurse is using the therapeutic communication technique of restatement. Restatement involves repeating the main idea of what the client has said. The nurse uses this technique to communicate that the client's statement has been heard and understood.

A 10-week, prenuptial counseling group composed of five couples is terminating. At the last group meeting, a nurse notices that the two most faithful and participative couples are absent. When considering concepts of group development, what might explain this behavior? A. They are experiencing problems with termination, leading to feelings of abandonment. B. They did not think any new material would be covered at the last session. C. They were angry with the leader for not extending the length of the group. D. They were bored with the material covered in the group.

ANS: A The nurse should determine that the clients' absence from the final group meeting may indicate that they are experiencing problems with termination. The termination phase of group development may elicit feelings of abandonment and anger. Successful termination may help members develop skills to cope with future unrelated losses.

Which client response should a nurse expect during the working phase of the nurse-client relationship? A. The client gains insight and incorporates alternative behaviors. B. The client and nurse establish rapport and mutually develop treatment goals. C. The client explores feelings related to reentering the community. D. The client explores personal strengths and weaknesses that impact behaviors.

ANS: A The nurse should expect that the client would gain insight and incorporate alternative behaviors during the working phase of the nurse-client relationship. The client may also overcome resistance, problem solve, and continually evaluate progress toward goals.

A newly admitted client asks, "Why do we need a unit schedule? I'm not going to these groups. I'm here to get some rest." Which is the most appropriate nursing reply? A. "Group therapy provides the opportunity to learn and practice new coping skills." B. "Group therapy is mandatory. All clients must attend." C. "Group therapy is optional. You can go if you find the topic helpful and interesting." D. "Group therapy is an economical way of providing therapy to many clients concurrently."

ANS: A The nurse should explain to the client that the purpose of group therapy is to learn and practice new coping skills. A basic assumption of milieu therapy is that every interaction, including group therapy, is an opportunity for therapeutic intervention.

A client diagnosed with depression and substance abuse has an altered sleep pattern and demands that a psychiatrist prescribe a sedative. Which rationale explains why a nurse should encourage the client to first try nonpharmacological interventions? A. Sedative-hypnotics are potentially addictive and will lose their effectiveness due to tolerance. B. Sedative-hypnotics are expensive and have numerous side effects. C. Sedative-hypnotics interfere with necessary REM (rapid eye movement) sleep. D. Sedative-hypnotics are not as effective to promote sleep as antidepressant medications.

ANS: A The nurse should recommend nonpharmacological interventions to this client because sedative-hypnotics are potentially addictive and will lose their effectiveness due to tolerance. The effects of central nervous system depressants are additive with one another and are capable of producing physiological and psychological dependence.

A nurse understands that "financial and legal difficulties" are to Axis IV of the DSM-IV-TR as "bipolar disorder" is to which axis? A. Axis I B. Axis II C. Axis III D. Axis V

ANS: A The nurse should understand that clinical disorders, such as bipolar disorder, should be included in Axis I of the DSM-IV-TR diagnosis. The DSM-IV-TR uses a multiaxial system in which each axis refers to a different domain of information. Axis I should include clinical disorders and other conditions that may be a focus of clinical attention. Axis II should include information about personality disorders and mental retardation. Axis III should include information about general medication conditions. Axis IV should include information on the client's psychosocial and environmental problems. Axis V provides a Global Assessment of Functioning (GAF).

During a community meeting, a nurse encourages clients to present unit problems and discuss possible solutions. Which type of leadership style is the nurse demonstrating? A. Democratic B. Autocratic C. Laissez-faire D. Bureaucratic

ANS: A The nurse who encourages clients to present problems and discuss solutions is demonstrating a democratic leadership style. Democratic leaders share information with group members and promote decision making by the members of the group. The leader provides guidance and expertise as needed.

To promote self-reliance, how should a psychiatric nurse best conduct medication administration? A. Encourage clients to request their medications at the appropriate times. B. Refuse to administer medications unless clients request them at the appropriate times. C. Allow the clients to determine appropriate medication times. D. Take medications to the clients' bedside at the appropriate times.

ANS: A The psychiatric nurse promoting self-reliance would encourage clients to request their medications at the appropriate times. Nurses are responsible for the management of medication administration on inpatient psychiatric units; however, nurses must work with clients to foster independence and provide experiences that would foster increased self-esteem.

Which example of a therapeutic communication technique would be effective in the planning phase of the nursing process? A. "We've discussed past coping skills. Let's see if these coping skills can be effective now." B. "Please tell me in your own words what brought you to the hospital." C. "This new approach worked for you. Keep it up." D. "I notice that you seem to be responding to voices that I do not hear."

ANS: A This is an example of the therapeutic communication technique of formulating a plan of action. By the use of this technique, the nurse can help the client plan in advance to deal with a stressful situation which may prevent anger and/or anxiety from escalating to an unmanageable level.

Which of the following individuals are communicating a message? (Select all that apply.) A. A mother spanking her son for playing with matches B. A teenage boy isolating himself and playing loud music C. A biker sporting an eagle tattoo on his biceps D. A teenage girl writing, "No one understands me" E. A father checking for new e-mail on a regular basis

ANS: A, B, C, D The nurse should determine that spanking, isolating, getting tattoos, and writing are all ways in which people communicate messages to others. It is estimated that about 70% to 90% of communication is nonverbal.

Which of the following nursing statements exemplify the cognitive process that must be completed by a nurse prior to caring for clients diagnosed with substance-abuse disorders? (Select all that apply.) A. "I am easily manipulated and need to work on this prior to caring for these clients." B. "Because of my father's alcoholism, I need to examine my attitude toward these clients." C. "I need to review the side effects of the medications used in the withdrawal process." D. "I'll need to set boundaries to maintain a therapeutic relationship." E. "I need to take charge when dealing with clients diagnosed with substance disorders."

ANS: A, B, D The nurse should examine personal bias and preconceived negative attitudes prior to caring for clients diagnosed with substance-abuse disorders. A deficit in this area may affect the nurse's ability to establish therapeutic relationships with these clients.

hich information should a nurse expect to be coded on a DSM-IV-TR axis? (Select all that apply.) A. Physical disorders B. Psychosocial stressors C. Treatment recommendations D. Global assessment of functioning E. Past suicide attempts

ANS: A, B, D The nurse should expect to see physical disorders, psychosocial stressors, and the Global Assessment of Function (GAF) coded on the DSM-IV-TR. The DSM-IV-TR is multidimensional and contains information that may help guide the course of treatment. Previous suicide attempts are symptoms of a clinical disorder such as bipolar disorder or major depressive disorder, and these diagnoses, not symptoms, would be included on Axis I.

Which of the following observed client behaviors would lead a nurse to evaluate a member as assuming a maintenance group role? (Select all that apply.) A. A client decreases conflict within the group by encouraging compromise. B. A client offers recognition and acceptance of others. C. A client outlines the task at hand and proposes solutions. D. A client listens attentively to group interaction. E. A client uses the group to gain sympathy from others.

ANS: A, B, D The nurse should identify clients who decrease conflict within the group, offer recognition and acceptance of others, and listen attentively to group interaction as assuming a maintenance group role. There are member roles within each group. Maintenance roles include the compromiser, the encourager, the follower, the gatekeeper, and the harmonizer.

A mother who has a history of chronic heroin use has lost custody of her children due to abuse and neglect. She has been admitted to an inpatient substance-abuse program. Which client statement should a nurse associate with a positive prognosis for this client? A. "I'm not going to use heroin ever again. I know I've got the willpower to do it this time." B. "I cannot control my use of heroin. It's stronger than I am." C. "I'm going to get all my children back. They need their mother." D. "Once I deal with my childhood physical abuse, recovery should be easy."

ANS: B A positive prognosis is more likely when a client admits that he or she is addicted to a substance and has a loss of control. One of the first steps in accepting treatment is for the client to admit powerlessness over the substance.

A student nurse tells the instructor, "I'm concerned that when a client asks me for advice I won't have a good solution." Which should be the nursing instructor's best response? A. "It's scary to feel put on the spot by a client. Nurses don't always have the answer." B. "Remember, clients, not nurses, are responsible for their own choices and decisions." C. "Just keep the client's best interests in mind and do the best that you can." D. "Set a goal to continue to work on this aspect of your practice."

ANS: B Giving advice tells the client what to do or how to behave. It implies that the nurse knows what is best and that the client is incapable of any self-direction. It discourages independent thinking.

In assessing a client diagnosed with polysubstance abuse, the nurse should recognize that withdrawal from which substance may require a life-saving emergency intervention? A. Dextroamphetamine (Dexedrine) B. Diazepam (Valium) C. Morphine (Astramorph) D. Phencyclidine (PCP)

ANS: B If large doses of central nervous system (CNS) depressants (like Valium) are repeatedly administered over a prolonged duration, a period of CNS hyperexcitability occurs on withdrawal of the drug. The response can be quite severe, even leading to convulsions and death.

Which is the priority nursing intervention for a client admitted for acute alcohol intoxication? A. Darken the room to reduce stimuli in order to prevent seizures. B. Assess aggressive behaviors in order to intervene to prevent injury to self or others. C. Administer lorazepam (Ativan) to reduce the rebound effects on the central nervous system. D. Teach the negative effects of alcohol on the body.

ANS: B Symptoms associated with the syndrome of alcohol intoxication include but are not limited to aggressiveness, impaired judgment, impaired attention, and irritability. Safety is a nursing priority in this situation.

A mother rescues two of her four children from a house fire. In the emergency department, she cries, "I should have gone back in to get them. I should have died, not them." What is the nurse's best response? A. "The smoke was too thick. You couldn't have gone back in." B. "You're feeling guilty because you weren't able to save your children." C. "Focus on the fact that you could have lost all four of your children." D. "It's best if you try not to think about what happened. Try to move on."

ANS: B The best response by the nurse is, "You're experiencing feelings of guilt because you weren't able to save your children." This response utilizes the therapeutic communication technique of reflection which identifies a client's emotional response and reflects these feelings back to the client so that they may be recognized and accepted.

What is the main goal of the working phase of the nurse-client therapeutic relationship? A. Role modeling to improve interaction with others B. Resolution of the client's problems C. Using therapeutic communication to clarify perceptions D. Helping the client access outpatient treatment

ANS: B The goal of the working phase of the nurse-client therapeutic relationship is to resolve client problems by promoting behavioral change.

Which situation exemplifies both assault and battery? A. The nurse becomes angry, calls the client offensive names, and withholds treatment. B. The nurse threatens to "tie down" the client and then does so against the client's wishes. C. The nurse hides the client's clothes and medicates the client to prevent elopement. D. The nurse restrains the client without just cause and communicates this to family.

ANS: B The nurse in this situation has committed both assault and battery. Assault refers to an action that results in fear and apprehension that the person will be touched without consent. Battery is the touching of another person without consent.

A hungry, homeless client, diagnosed with schizophrenia, refuses to participate in an admission interview. A nurse streamlines the assessment, verbally assures safety, and provides a warm meal. What is the nurse promoting by these actions? A. Sympathy B. Trust C. Veracity D. Manipulation

ANS: B The nurse is promoting trust by streamlining the assessment, assuring safety, and providing a warm meal. Trust implies a feeling of confidence in that a person is reliable and sincere and has integrity and veracity. Trustworthiness is demonstrated though nursing interventions that convey a sense of warmth and caring to the client.

A nurse evaluates a client's patient-controlled analgesia (PCA) pump and notices 100 attempts within a 30-minute period. Which is the best rationale for assessing this client for substance dependence? A. Narcotic pain medication is contraindicated for all clients with active substance-abuse problems. B. Clients who are dependent on alcohol or benzodiazepines may have developed cross-tolerance to analgesics and require increased doses to achieve effective pain control. C. There is no need to assess the client for substance dependence. There is an obvious PCA malfunction. D. The client is experiencing symptoms of withdrawal and needs to be accurately assessed for lorazepam (Ativan) dosage.

ANS: B The nurse should assess the client for substance dependence because clients who are dependent on alcohol or benzodiazepines may have developed cross-tolerance to analgesics, and require increased doses to achieve effective pain control. Cross-tolerance occurs when one drug lessened the client's response to another drug.

During a therapeutic group, a client talks about personal accomplishments in an effort to gain attention. Which group role, assumed by this client, should the nurse identify? A. The task role of gatekeeper B. The individual role of recognition seeker C. The maintenance role of dominator D. The task role of elaborator

ANS: B The nurse should evaluate that the client is assuming the individual role of the recognition seeker. Other individual roles include the aggressor, the blocker, the dominator, the help seeker, the monopolizer, and the seducer.

If an individual is "two-faced," which characteristic essential to the development of a therapeutic relationship should a nurse identify as missing? A. Respect B. Genuineness C. Sympathy D. Rapport

ANS: B The nurse should identify that genuineness is missing in the relationship. Genuineness refers to the nurse's ability to be open and honest and maintain congruence between what is felt and what is communicated. When a nurse fails to bring genuineness to the relationship, trust cannot be established.

A nurse maintains an uncrossed arm and leg posture. This nonverbal behavior is reflective of which letter of the SOLER acronym for active listening? A. S B. O C. L D. E E. R

ANS: B The nurse should identify that maintaining an uncrossed arm and leg posture is nonverbal behavior that reflects the "O" in the active-listening acronym SOLER. The acronym SOLER includes sitting squarely facing the client (S), open posture when interacting with the client (O), leaning forward toward the client (L), establishing eye contact (E), and relaxing (R).

Which situation should a nurse identify as an example of an autocratic leadership style? A. The president of Sigma Theta Tau assigns members to committees to research problems. B. Without faculty input, the dean mandates that all course content be delivered via the Internet. C. During a community meeting, a nurse listens as clients generate solutions. D. The student nurses' association advertises for candidates for president.

ANS: B The nurse should identify that mandating decisions without consulting the group is considered an autocratic leadership style. Autocratic leadership increases productivity but often reduces morale and motivation due to lack of member input and creativity.

Which client should a nurse identify as a potential candidate for involuntary commitment? A. A client living under a bridge in a cardboard box B. A client threatening to commit suicide C. A client who never bathes and wears a wool hat in the summer D. A client who eats waste out of a garbage can

ANS: B The nurse should identify the client threatening to commit suicide as eligible for involuntary commitment. The suicidal client who refuses treatment is a danger to self and requires emergency treatment.

According to Maslow's hierarchy of needs, which situation on an in-patient psychiatric unit would require priority intervention by a nurse? A. A client rudely complaining about limited visiting hours B. A client exhibiting aggressive behavior toward another client C. A client stating that no one cares D. A client verbalizing feelings of failure

ANS: B The nurse should immediately intervene when a client exhibits aggressive behavior toward another client. Safety and security are considered lower-level needs according to Maslow's hierarchy of needs and must be fulfilled before other higher-level needs can be met. Clients who complain, have feelings of failure, or state that no one cares are struggling with higher-level needs such as the need for love and belonging or the need for self-esteem.

What is the best rationale for including the client's family in therapy within the inpatient milieu? A. To structure a program of social and work-related activities B. To facilitate discharge from the hospital C. To provide a concrete demonstration of caring D. To encourage the family to model positive behaviors

ANS: B The nurse should include the client's family in therapy within the inpatient milieu to facilitate discharge from the hospital. Family members are invited to participate in some therapy groups and to share meals with the client in the communal dining room. Family involvement may also serve to prevent the client from becoming too dependent on the therapeutic environment.

A client has a history of excessive drinking which has led to multiple driving under the influence (DUI) arrests. The client states, "I work hard to provide for my family. I don't see why I can't drink to relax." The nurse recognizes the use of which defense mechanism? A. Projection B. Rationalization C. Regression D. Sublimation

ANS: B The nurse should recognize that the client is using rationalization, a common defense mechanism. The client is attempting to make excuses and create logical reasons to justify unacceptable feelings or behaviors.

Which nursing statement about the concept of psychoses is most accurate? A. "Individuals experiencing psychoses are aware that their behaviors are maladaptive." B. "Individuals experiencing psychoses experience little distress." C. "Individuals experiencing psychoses are aware of experiencing psychological problems." D. "Individuals experiencing psychoses are based in reality."

ANS: B The nurse should understand that the client with psychoses experiences little distress due to his or her lack of awareness of reality. The client with psychoses is unaware that his or her behavior is maladaptive or that he or she has a psychological problem.

Which nursing statement about the concept of neuroses is most accurate? A. "An individual experiencing neurosis is unaware that he or she is experiencing distress." B. "An individual experiencing neurosis feels helpless to change his or her situation." C. "An individual experiencing neurosis is aware of psychological causes of his or her behavior." D. "An individual experiencing neurosis has a loss of contact with reality."

ANS: B The nurse should understand that the concept of neuroses includes the following characteristics: The client feels helpless to change his or her situation, the client is aware that he or she is experiencing distress, the client is aware the behaviors are maladaptive, the client is unaware of the psychological causes of the distress, and the client experiences no loss of contact with reality.

Which phase of the nurse-client relationship begins when the individuals first meet and is characterized by an agreement to continue to meet and work on setting client-centered goals? A. Preinteraction B. Orientation C. Working D. Termination

ANS: B The orientation phase is when the individuals first meet and is characterized by an agreement to continue to meet and work on setting client-centered goals. There are four phases of relationship development: preinteraction, orientation, working, and termination.

Which statement reflects a student nurse's accurate understanding of the concepts of mental health and mental illness? A. "The concepts are rigid and religiously based." B. "The concepts are multidimensional and culturally defined." C. "The concepts are universal and unchanging." D. "The concepts are unidimensional and fixed."

ANS: B The student nurse should understand that mental health and mental illness are multidimensional and culturally defined. It is important for nurses to be aware of cultural norms when evaluating the client's mental state.

Which of the following are accurate descriptors of a therapeutic community? (Select all that apply.) A. The unit schedule includes unlimited free time for personal reflection. B. Unit responsibilities are assigned according to client capabilities. C. A flexible schedule is determined by client needs. D. The individual is the sole focus of therapy. E. A democratic form of government exists.

ANS: B, E In a therapeutic community, the unit responsibilities are assigned according to client capability and a democratic form of government exists. Therapeutic communities are structured and provide therapeutic interventions that focus on communication and relationship-development skills.

The nursing staff is discussing the concept of competency. Which information about competency should a nurse recognize as true? A. Competency is determined with a client's compliance with treatment. B. Refusal of medication can initiate an incompetency hearing leading to forced medications. C. A competent client has the ability to make reasonable judgments and decisions. D. Competency is a medical determination made by the client's physician.

ANS: C A competent individual's cognition is not impaired to an extent that would interfere with decision making.

A recovering alcoholic relapses and drinks a glass of wine. The client presents in the emergency department (ED) experiencing severe throbbing headache, tachycardia, flushed face, dyspnea, and continuous vomiting. What may these symptoms indicate to the ED nurse? A. Alcohol poisoning B. Cardiovascular accident (CVA) C. A reaction to disulfiram (Antabuse) D. A reaction to tannins in the red wine

ANS: C Ingestion of alcohol while disulfiram is in the body results in a syndrome of symptoms that can produce a good deal of discomfort for the individual. Symptoms may include but are not limited to flushed skin, throbbing in the head and neck, respiratory difficulty, dizziness, nausea and vomiting, confusion, hypotension, and tachycardia.

An employee uses the defense mechanism of displacement when the boss openly disagrees with suggestions. What behavior would be expected from this employee? A. The employee assertively confronts the boss B. The employee leaves the staff meeting to work out in the gym C. The employee criticizes a coworker D. The employee takes the boss out to lunch

ANS: C The client using the defense mechanism of displacement would criticize a coworker after being confronted by the boss. Displacement refers to transferring feelings from one target to a neutral or less-threatening target.

A client diagnosed with dependant personality disorder states, "Do you think I should move from my parent's house and get a job?" Which nursing response is most appropriate? A. "It would be best to do that in order to increase independence." B. "Why would you want to leave a secure home?" C. "Let's discuss and explore all of your options." D. "I'm afraid you would feel very guilty leaving your parents."

ANS: C The most appropriate response by the nurse is, "Let's discuss and explore all of your options." In this example, the nurse is encouraging the client to formulate ideas and decide independently the appropriate course of action.

Which therapeutic communication technique is being used in this nurse-client interaction? Client: "When I am anxious, the only thing that calms me down is alcohol." Nurse: "Other than drinking, what alternatives have you explored to decrease anxiety?" A. Reflecting B. Making observations C. Formulating a plan of action D. Giving recognition

ANS: C The nurse is using the therapeutic communication technique of formulating a plan of action to help the client explore alternatives to drinking alcohol. The use of this technique, rather than direct confrontation regarding the client's poor coping choice, may serve to prevent anger or anxiety from escalating.

During a therapeutic group, which nursing action demonstrates a laissez-faire leadership style? A. The nurse mandates that all group members reveal an embarrassing personal situation. B. The nurse asks for a show of hands to determine group topic preference. C. The nurse sits silently as the group members stray from the assigned topic. D. The nurse shuffles through papers to determine the facility policy on length of group.

ANS: C The nurse leader who sits silently and allows group members to stray from the assigned topic is demonstrating a laissez-faire leadership style. This style allows group members to do as they please with no direction from the leader. Group members often become frustrated and confused in reaction to a laissez-faire leadership style.

A nursing instructor is presenting content on the provisions of the nurse practice act as it relates to their state. Which student statement indicates a need for further instruction? A. "The nurse practice act provides a list of definitions of important terms including the definition of nursing." B. "The nurse practice act lists education requirements for licensure and reciprocity." C. "The nurse practice act contains detailed statements that describe the scope of practice for registered nurses (RNs)." D. "The nurse practice act lists the general authority and powers of the state board of nursing."

ANS: C The nurse practice act contains broad, not detailed, statements that describe the scope of practice for various levels of nursing (APN, RN, LPN), not just for the RN. This student statement indicates a need for further instruction.

Upon admission for symptoms of alcohol withdrawal a client states, "I haven't eaten in 3 days." Assessment reveals BP 170/100 mm Hg, P 110, R 28, and T 97F (36C) with dry skin, dry mucous membranes, and poor skin turgor. What should be the priority nursing diagnosis? A. Knowledge deficit B. Fluid volume excess C. Imbalanced nutrition: less than body requirements D. Ineffective individual coping

ANS: C The nurse should assess that the priority nursing diagnosis is imbalanced nutrition: less than body requirements. The client is exhibiting signs and symptoms of malnutrition as well as alcohol withdrawal. The nurse should consult a dietitian, restrict sodium intake to minimize fluid retention, and provide small, frequent feedings of nonirritating foods.

During an intake assessment, a nurse asks both physiological and psychosocial questions. The client angrily responds, "I'm here for my heart, not my head problems." Which is the nurse's best response? A. "It's just a routine part of our assessment. All clients are asked these same questions." B. "Why are you concerned about these types of questions?" C. "Psychological factors, like excessive stress, have been found to affect medical conditions." D. "We can skip these questions, if you like. It isn't imperative that we complete this section."

ANS: C The nurse should attempt to educate the client on the negative effects of excessive stress on medical conditions. It is not appropriate to skip either physiological or psychosocial questions as this would lead to an inaccurate assessment.

A nursing instructor is teaching about psychodrama, a specialized type of therapeutic group. Which student statement indicates that further teaching is necessary? A. "Psychodrama provides a safe setting in which to discuss painful issues." B. "In psychodrama, the client is the protagonist." C. "In psychodrama, the client observes actor interactions from the audience." D. "Psychodrama facilitates resolution of interpersonal conflicts."

ANS: C The nurse should educate the student that in psychodrama the client plays the role of himself or herself in a life-situation scenario and is called the protagonist. During psychodrama, the client does not observe interactions from the audience. Other group members perform the role of the audience and discuss the situation they have observed, offer feedback, and express their feelings. Leaders of psychodrama must have specialized training to become a psychodramatist.

A fourth-grade boy teases and makes jokes about a cute girl in his class. This behavior should be identified by a nurse as indicative of which defense mechanism? A. Displacement B. Projection C. Reaction formation D. Sublimation

ANS: C The nurse should identify that the boy is using reaction formation as a defense mechanism. Reaction formation is the attempt to prevent undesirable thoughts from being expressed by expressing opposite thoughts or behaviors. Displacement refers to transferring feelings from one target to another. Rationalization refers to making excuses to justify behavior. Projection refers to the attribution of unacceptable feelings or behaviors to another person. Sublimation refers to channeling unacceptable drives or impulses into more constructive, acceptable activities.

A nurse is performing a mental health assessment on an adult client. According to Maslow's hierarchy of needs, which client action would demonstrate the highest achievement in terms of mental health? A. Maintaining a long-term, faithful, intimate relationship B. Achieving a sense of self-confidence C. Possessing a feeling of self-fulfillment and realizing full potential D. Developing a sense of purpose and the ability to direct activities

ANS: C The nurse should identify that the client who possesses a feeling of self-fulfillment and realizes his or her full potential has achieved self-actualization, the highest level on Maslow's hierarchy of needs.

A client tells the nurse, "I feel bad because my mother does not want me to return home after I leave the hospital." Which nursing response is therapeutic? A. "It's quite common for clients to feel that way after a lengthy hospitalization." B. "Why don't you talk to your mother? You may find out she doesn't feel that way." C. "Your mother seems like an understanding person. I'll help you approach her." D. "You feel that your mother does not want you to come back home?"

ANS: D This is an example of the therapeutic communication technique of restatement. Restatement is the repeating of the main idea that the client has verbalized. This lets the client know whether or not an expressed statement has been understood and gives him or her the chance to continue, or clarify if necessary.

b Fraudulent documentation may also be grounds for discipline by the state board of nursing. Referring the matter to the charge nurse will allow observance of hospital policy while ensuring that documentation occurs. Notifying the health care provider would be unnecessary when the charge nurse can resolve the problem.

After leaving work, a nurse realizes documentation of administration of a PRN medication was omitted. This off-duty nurse phones the unit and tells the nurse, "Please document administration of the medication for me; I forgot to do it. My password is alpha1." The nurse receiving the call should:

d Breach of nurse-patient confidentiality does not pose a legal dilemma for nurses in these circumstances because a team approach to delivery of psychiatric care presumes communication of patient information to other staff members to develop treatment plans and outcome criteria. The patient should know that the team may have to warn the father of the risk for harm.

An adolescent hospitalized after a violent physical outburst tells the nurse, "I'm going to kill my father, but you can't tell anyone." Select the nurse's best response. a. "You're right. Federal law requires me to keep that information private." b. "Those kinds of thoughts will make your hospitalization longer." c. "You really should share this thought with your psychiatrist." d. "I am obligated to that share information with the treatment team."

d Caring evidences empathetic understanding as well as competency. It helps change pain and suffering into a shared experience, creating a human connection that alleviates feelings of isolation. The distracters give examples of statements that demonstrate advocacy or giving advice.

An informal group of patients discusses their perceptions of nursing care. Which comment best indicates a patient perceived the nurse was caring? "My nurse: a. always asks me which type of juice I want to help me swallow my medication." b. explained my treatment plan to me and asked for my ideas about how to make it better." c. told me that if I take all the medicines the doctor prescribes, then I will get discharged soon." d. spends time listening to me talk about my problems. That helps me feel like I'm not alone."

a The nurse must consider the meaning, timing, and value of the gift. In this instance, the nurse should accept the patient's expression of gratitude.

As a nurse discharges a patient, the patient gives the nurse a card of appreciation made in an arts and crafts group. Select the nurse's best action. a. Recognize the patient's thoughtfulness. Express appreciation and accept the card. b. Inform the patient that accepting gifts violates policies of the facility. Decline the card. c. Acknowledge the patient's transition through the termination phase, but decline the card. d. Accept the card. Invite the patient to return to participate in other arts and crafts groups.

As a patient and nurse move into the working stage of a therapeutic relationship, the nurse's most beneficial statement is: "Of the problems we have discussed so far, which ones would you most like to work on?"

Complete this goal statement for a newly admitted patient. "By the end of the orientation stage of the therapeutic relationship, the patient will demonstrate: increased self-responsibility."

What do all third party payers insist on before considering reimbursement?

DSM diagnosis

d

During the first interview with a restless young man, the nurse notices that he does not make eye contact throughout most of the interview. It can correctly be assumed that a. he is not to be trusted in what he says because he is evasive b. he is really feeling sad and can't look the nurse in the eye c. he is shy and the nurse must move slowly d. more data is needed to draw a conclusion

d The data presented are insufficient to draw a conclusion. The nurse must continue to gather information.

During the first interview, a nurse notices that a patient does not make eye contact. Which analysis is correct? a. The patient is not truthful and is demonstrating evasive behavior. b. The patient is shy. The nurse should progress slowly. c. The patient feels sad and cannot look at the nurse. d. More information is needed to draw a conclusion.

Which scientist made huge contributions to the classification of mental disorders?

Emil Kraepelin

Who coined the phrase schizophrenia?

Eugene Bleuler

Who was the first psychiatric nursing theorist?

Hildegarde Paplau

The ___ is the personality process that wants to experience only pleasure and is impulsive and without morals

Id

c

In a session J cries as the nurse explores her relationship with her deceased mother. J sobs, "I shouldn't be blubbering like this." A statement by the nurse that will hinder communication is a. "The relationship with your mother is very painful for you." b. "I can see that you feel sad about this situation." c. "Why do you think you are feeling so upset?" d. "Crying is a way of expressing the hurt you're experiencing."

d

In a treatment team planning meeting a nurse states her concern about whether the staff is behaving ethically in using restraint to prevent one client from engaging in self-mutilative behavior when the care plan for another self-mutilating client calls for one-on-one supervision. The ethical principle that should govern the situation is a. Beneficence b. Autonomy c. Fidelity d. Justice

a

K, a college student who usually gets As, has received a C on a difficult examination. He runs to the college health service. He is disorganized and only partially coherent. In order to reduce K's level of anxiety as quickly as possible, the nurse should first a. stay with him b. tell him that help will come c. question him to discover the events that led to his de-compensation d. tell him he will not be allowed to hurt anyone

d

L is experiencing considerable stress. She is in a work environment in which her boss treats her "like a doormat." She states he thinks nothing of demanding that she stay overtime and work on Saturdays whenever he is "in the mood to work." A healthy coping strategy the nurse might suggest for L is a. resigning the position b. starting individual short term therapy c. relying on the support of her family d. employing assertiveness techniques

c Operant conditioning involves giving positive reinforcement for a desired behavior. Treats are rewards and reinforce speech through positive reinforcement.

Operant conditioning is part of the treatment plan to encourage speech in a child who is nearly mute. Which technique applies? a. Encourage the child to observe others talking. b. Include the child in small group activities. c. Give the child a small treat for speaking. d. Teach the child relaxation techniques.

What case gave nonviolent patients the right to refuse medication?

Rogers vs. Orkin

a Nurses are professionally bound to uphold the American Nurses Association standards of practice regardless of lesser standards established by a health care agency or a state. Conversely, if the agency standards are higher than the American Nurses Association standards of practice, the agency standards must be upheld. The courts may seek to establish the standard of care through the use of expert witnesses when the issue is clouded.

Several nurses are concerned that agency policies related to restraint and seclusion practices are inadequate. Which understanding about the relationship of substandard institutional policies and individual nursing practice should guide nursing practice? a. The policies do not absolve an individual nurse of the responsibility to practice according to professional standards of nursing care. b. Agency policies are considered the legal standard by which a professional nurse must act and therefore override other standards of care. c. In an institution with substandard policies, the nurse has a responsibility to inform the supervisor and leave the premises without delay. d. Interpretation of policies by the judicial system is rendered on an individual basis and therefore cannot be predicted.

d

T is an involuntary client on a psychiatric inpatient unit. He asks the nurse for a 5 day release for discharge. The best response for the nurse to make would be a. "I can't give you those forms without your doctor's knowledge." b. "Here is the number for the guardianship and advocacy commission. They will send you the forms." c. "Because you are an involuntary patient you are committed here for 30 days" d. "I will get you the form for you to fill out:

a

T states that before taking an examination he feels a heightened sense of awareness and a sense of restlessness. What nursing intervention would be most suitable for assisting T? a. Explain his symptoms as resulting from mild anxiety and discuss the helpful aspects of mild anxiety. b. Advise T to discuss his experience with a physician or psychologist. c. Offer to obtain an order for an anxiolytic that T can take when necessary. d. Listen without comment.

a Summarizing and evaluating progress help validate the experience for the patient and the nurse and facilitate closure. Termination must be discussed; avoiding discussion by spending little time with the patient promotes feelings of abandonment. Successful termination requires that the relationship be brought to closure without the possibility of dependency-producing ongoing contact.

Termination of a therapeutic nurse-patient relationship has been successful when the nurse: a. discusses with the patient changes that happened during the relationship and evaluates outcomes. b. gives the patient the nurse's cell phone number and permission to call after discharge. c. avoids upsetting the patient by shifting focus to other patients before the patient's discharge. d. offers to meet the patient for conversation after discharge.

d

The nurse caring for an angry manipulative client finds himself feeling angry with the client. The nurse should initially: a. refuse to care for the patient b. let the client know how he feels c. tell the nurse manager to assign the client to another nurse d. deal with his feelings in a supervisory session

d Many of the most prevalent and disabling mental disorders have strong biological influences. Genetics are only one part of biological factors. Empathy does not address increasing the spouse's level of knowledge about the cause of the disorder. The other distracters are not established facts.

The spouse of a patient with schizophrenia says, "I don't understand how nurturing or toilet training in childhood has anything to do with this incredibly disabling illness." Which response by the nurse will best help the spouse understand this disorder? a. "This illness is the result of genetic factors." b. "Psychological stress is at the root of most mental disorders." c. "It must be frustrating for you that your spouse is sick so much of the time." d. "New findings show that this condition more likely has biological rather than psychological origins."

A nurse plans to teach a group of patients the basics of the change process. How should the elements be sequenced for the presentation?

This sequence proceeds logically from assessment of the problem to analysis of the problem to determining that change is necessary to testing new behaviors and evaluating their efficacy.

b The practice of psychiatric nursing requires a different set of skills than medical-surgical nursing, though there is substantial overlap. Psychiatric nurses must be able to help patients with medical as well as mental health problems, reflecting the holistic perspective these nurses must have. Nurse-patient ratios and workloads in psychiatric settings have increased, just like other specialties. Psychiatric nursing involves clinical practice, not just documentation. Psychosocial pain and suffering is as real as physical.

Two nursing students discuss their career plans after graduation. One student wants to enter psychiatric nursing. The other asks, "Why would you want to be a psychiatric nurse? The only thing they do is talk. You'll lose all your skills." Select the best response. a. "Psychiatric nurses practice in safer environments than other specialties. Nurse-to-patient ratios must be better because of the nature of the patients' problems." b. "Psychiatric nurses use complex communication skills as well as critical thinking to solve multidimensional problems. I am challenged by those situations." c. "I think I will be good in the mental health field. I did not like clinical rotations in school, so I do not want to continue them after I graduate." d. "Psychiatric nurses do not have to deal with as much pain and suffering as medical-surgical nurses do. That appeals to me."

a

What is the legal significance of the nurse's action when a client verbally refuses his medication and the nurse gives it over his objection? a. The nurse can be charged with battery. b. The nurse can be charged with negligence. c. The nurse can be charged with malpractice. d. No charges can be brought against the nurse.

d Staying with a crying patient offers support and shows positive regard. The distracters describe tasks, not necessarily with positive regard, and the nurse's efforts to remain self-aware.

Which action by a nurse shows positive regard? a. Making rounds according to the daily assignment. b. Administering daily medication as prescribed. c. Examining own feelings about a patient. d. Staying with a patient who is crying.

b

Which of the following is a description of the least restrictive alternative? a. Allowing the client to sign a 5 day release for discharge b. Offering the client a by mouth dose of medication instead of an injection c. Signing the petition, cert, and then another cert within 24 hours d. Placing the client in seclusion rather than restraints

d

Which statement given below would be an example showing that Nurse G is using an open ended exploring question with her patient: a. "Do you think the medications are helping you?" b. "When were you born?" c. "Is there mental illness in your family?" d. "Tell me more about what was going on before your suicide attempt."

a Empathy permits the nurse to see an event from the patient's perspective, understand the patient's feelings, and communicate this to the patient. The distracters focus on the nurse's feelings rather than the patient's, promote patient dependence, and belittle the patient's perspective.

Which statement shows a nurse has empathy for a patient who made a suicide attempt? a. "You must have been very upset to do what you did today." b. "It makes me sad to see you going through such a difficult experience." c. "If you tell me what is troubling you, I can help you solve your problems." d. "Suicide is a drastic solution to a problem that may not be such a serious matter."

b A contract emphasizes that the nurse works with the patient rather than doing something for the patient. "Working with" is a process that suggests each party is expected to participate and share responsibility for outcomes.

Why should a nurse introduce the matter of a contract during the first session with a new patient? Contracts: a. specify what the nurse will do for the patient. b. spell out participation and responsibilities of both parties. c. are indicative of the feeling tone established between the participants. d. are binding and prevent either party from prematurely ending the relationship.

The Missouri State Board of Nursing is an example of which branch of law?

administrative law

What is the most prevalent psychiatric disorder for those over 17 years of age?

anxiety

nursing process

apply nursing process to provide comprehensive nursing care, nursing process is the foundation of case management, nursing process is effective in all aspects of psychiatric mental health nursing, nursing process and psychotherapeutic management are the backbone of psychiatric nursing

____ is the deliberate threat coupled with the apparent ability to do physical harm to another

assault

___ is a placed of safety or sanctuary, a refuge, an institution for the care of mentally ill, often associated with mistreatment and callousness

asylum

____ led to wandering bands of "lunatics" and no one cared if they lived or died

banishment

Why is the M'Naghten criteria also known as the cognitive standard?

because it focuses on the knowledge of right and wrong

Why is it important to have a basic understanding of the DSM?

because it is the language of psychiatry

What benchmark was the period of enlightenment?

benchmark 1

The shift in focus from sanctuary to treatment is linked to which benchmark?

benchmark 2

In what benchmark were significant changes in the Diagnostic and Statistical Manual of Mental Disorders made?

benchmark 5

___ is failure to conform to or the departure from a required duty of care owed to a person

breach of duty

psychotherapeutic management model

clarifies and distinguishes the role of the psychiatric nurse, applications of psychotherapeutic interventions (3 dimensions), knowledge of psychopathology

____ is used when a patient who has legal competence to request treatment but refuses to do so

commitment (involuntary patients)

What must the nurse do when a patient makes a threat before taking action to ensure that the patient's rights are balanced with those of the third party?

communicate confidential patient communications with the clinical team

nurse patient relationship

communication skills, respect ad desire to help, understanding of mental mechanisms, adaptation styles, coping strategies, therapeutic intervention skills

The _____ raised public awareness of mental disorders and nursing textbooks began to provide more detailed information about psychopharmacology

decade of the brain

what are the major mental disorders that cause disability?

depression, schizophrenia, alcohol abuse, bipolar disorder

The Diagnostic and Statistical Manual of Mental Disorders (DSM) is also known as what?

diagnostic bible of psychiatry

Which duty can arise from a special relationship, such as the relationship between a nurse and a patient?

duty to care

The ___ is the personality process that focuses on reality, while striving to meet the needs of the id, and experiences anxiety and uses defense mechanisms for protection

ego

____ care usually lasts 48-72 hours and is for individuals who are dangerous to self or others or are gravely disabled by mental illness

emergency

In which period did psychiatric nursing have its origins?

enlightenment period

The test for ____ is whether anyone of ordinary prudence and intelligence should have anticipated the danger to another caused by his or her negligence

foreseeability

How many major benchmarks have contributed to the evolution of modern psychiatric care?

four

continuum of care treatment options

hospital based care, residential services, outpatient services, day treatment programs, self help groups, primary care

What is free association?

in a therapeutic context, saying anything that comes to mind

determining placement on the care continuum

initial contact, assess and refer to services that are least restrictive, most effective, most cost-conscious, risk assessment provides ongoing guidance to appropriate care level

_____ treatment is the area of psychiatric care from which most legal issues arise

involuntary

What occurred during benchmark 1?

it was initiated by the institution of humane care, which ultimately led to the asylum movement and, under the influence of Dorothea Dix, the development of the state hospital system in the US

Documentation that effective communication has occurred is a safeguard against ____

lawsuits

____ was also introduced during thee benchmark 3 period but was not used in the US until 1970

lithium

____ is used for persons who need prolonged psychiatric care, but refuse to seek help voluntarily. It typically lasts 90 days or longer

long-term commitment

continuum of care model

milieu management, nurse patient relationship, psychopharmacology

diagnostic statistical manual

official guideline for diagnosing psychiatric disorders, first published in 1952, DSM classifies disorders it does not classify people

Which nursing intervention will initially be most helpful for trust building with a suspicious patient? Keeping appointments and promises

patient shouts at a nurse who just entered the room, "You're an incompetent fool. Leave me alone. nurse's response should be based on which rationale? anger was created by a situation or significant person, not the nurse.

Therapies such as psychoanalysis, psychotherapy, and psychosurgery were developed during which period?

period of scientific study

The greatest impact in the care of the mentally ill of the past 50 years has resulted from what?

progress and improvement in the area of psychotropic drugs

why study the history

provides information on where we have been and how we have reached the understanding that we use to provide care today, assists in understanding the care provided today in the context of history, provides foundation for the book

____ is a reasonable, close, and causal connection or relationship exists between the defendant's negligent conduct and the resulting damages suffered by the plaintiff

proximate cause

patient hospitalization is most similar to which treatment option?

residential services

continuum of care

role of the nurse is to assess and direct or escort the patient to appropriate services, multidisciplinary team collaboration coordinates care, team may include professions, patients or consumers, families, representatives from insurance companies, nursing homes, group homes or medical clinical

The time of stay varies from state to state in _____ but can last up to 14 days

short-term observation and treatment

____ is the degree of skill, care, and knowledge ordinarily possessed and exercised by one who has a duty to care

standard of care

What is negligence?

the failure to do or NOT to do what a reasonably careful person would do under the same circumstances

What is gravely disabled?

the inability to provide food, clothing, and shelter for oneself because of mental illness

Under the Master-Servant rule who is responsible for the consequences of the act and for the adequate supervision of the assistant?

the licensed person who delegated the act

Historically, societies have neglected the rights of ____

the mentally ill

the role of the inpatient nurse on a behavioral health unit encompasses

therapeutic relationships, administering medications, and maintaining safety

psychopharmacology

therapeutic versus toxic dosage levels, use during pregnancy, use with older adults, side effects, interactions, side effects, interactions, patient teaching

What was done with mentally ill patients in the 1700's?

they were placed on display for the amusement of the paying public

What was considered a breakthrough medication as the first psychiatric medication?

thorazine

safety

trumps all other patient concerns

Axis __ is psychosocial stressors such as divorce, housing, and education issues

4

b

45. Which of the following anti-anxiety medications is considered non-addictive? a. valium b. buspirone c. klonopin d. xanax

d The patient is unable to maintain personal hygiene, oral intake, or verbal communication. The patient is a danger to self because of not eating. The distracters represent higher levels of functioning.

A patient is depressed, mute, and motionless. According to family members, the patient has refused to bathe or eat for a week. The patient's global assessment of functioning score is: a. 100 b. 50 c. 25 d. 10

Which patient behavior would require the most immediate limit setting? A patient shouts at a roommate, "You are perverted! You watched me undress."

A patient playing pool with another patient throws down the pool cue and begins swearing. The nurse should initially intervene by: suggesting a time-out in the patient's room.

Which is an example of an intentional tort? A. A nurse fails to assess a client's obvious symptoms of neuroleptic malignant syndrome. B. A nurse physically places an irritating client in four-point restraints. C. A nurse makes a medication error and does not report the incident. D. A nurse gives patient information to an unauthorized person.

ANS: B A tort is a violation of civil law in which an individual has been wronged and can be intentional or unintentional. A nurse who physically places an irritating client in restraints has touched the client without consent and has committed an intentional tort.

A client states, "You won't believe what my husband said to me during visiting hours. He has no right treating me that way." Which nursing response would best assess the situation that occurred? A. "Does your husband treat you like this very often?" B. "What do you think is your role in this relationship?" C. "Why do you think he behaved like that?" D. "Describe what happened during your time with your husband."

ANS: D This is an example of the therapeutic communication technique of exploring. The purpose of using exploring is to delve further into the subject, idea, experience, or relationship. This technique is especially helpful with clients who tend to remain on a superficial level of communication.

a

The nurse wishes to teach an alternative way of coping to a client experiencing severe anxiety. The nurse will first need to a. use measures designed to lower the client's anxiety b. determine the mode of learning preferred by the client c. devise outcomes and construct a teaching plan d. place the client in seclusion until the anxiety subsides

What is deinstitutionalization?

a shift in treatment location from large public hospitals to community settings, this process began in 1955 and federal actions supported it

Which axis is most often used?

axis 1

____ patients are when the patient, conservator, or therapist requests admission

voluntary

What percentage of homeless are children?

25%

d

4. The nurse must assess the mental health or mental illness of several new clients at the mental health clinic. Some of the traits of mental health include: a. Accurate appraisal of reality b. Ability to work and be productive c. Ability to control one's own behavior d. All of the above

b

44. Which of the following is considered a side effect of the anti-anxiety (anxiolytic) medication ativan? a. Decreased tolerance b. Increased sedation c. Increased cognitive function d. All of the above

The DSM-IV-TR provides how many axes to use in patient assessment?

5

A pt with a history of self-mutilation says to the nurse, "I want to stop hurting myself." What is the initial step of the problem-solving process to be taken toward resolution of a patient's problem?Describing the problem or situation

A patient says, "I went out drinking only one time last week. At least I'm trying to change." The nurse responds, "I appreciate your effort, but you agreed to abstain from alcohol completely." The nurse is preventing manipulation.

b It is the health care professional's duty to warn or notify an intended victim after a threat of harm has been made. Informing a potential victim of a threat is a legal responsibility of the health care professional. It is not a violation of confidentiality.

A patient with schizophrenia believes a local minister stirred evil spirits and threatens to bomb a local church. The psychiatrist notifies the minister. Select the answer with the correct rationale. The psychiatrist: a. released information without proper authorization. b. demonstrated the duty to warn and protect. c. violated the patient's confidentiality. d. avoided charges of malpractice.

a

A woman speaking of a rival for her husband's affection says in a gushy, syrupy voice, "She is a lovely person. I simply adore her." The woman may be employing a. Reaction formation b. Projection c. Denial d. Repression

According to Peplau, which nursing intervention is most appropriate when the nurse is functioning in the role of a surrogate? A. The nurse functions as a nurturing parent in order to build a trusting relationship. B. The nurse plays cards with a small group of clients. C. The nurse discusses childhood events that may affect personality development. D. The nurse provides a safe social environment.

ANS: A According to Peplau, when a client is acutely ill, he or she may incur the role of infant or child while the nurse is perceived as the mother surrogate.

A nurse is reviewing STAT laboratory data of a client presenting in the emergency department. At what minimum blood alcohol level should a nurse expect intoxication to occur? A. 50 mg/dL B. 100 mg/dL C. 250 mg/dL D. 300 mg/dL

ANS: B The nurse should expect that 100 mg/dL is the minimum blood alcohol level at which intoxication occurs. Intoxication usually occurs between 100 and 200 mg/dL. Death has been reported at levels ranging from 400 to 700 mg/dL.

Which is an example of the ego defense mechanism of regression? A. A mother blames the teacher for her child's failure in school. B. A teenager becomes hysterical after seeing a friend killed in a car accident. C. A woman wants to marry a man exactly like her beloved father. D. An adult throws a temper tantrum when he does not get his own way.

ANS: D Regression is the retreating to an earlier level of development and the comfort measures associated with that level of functioning.

When under stress, a client routinely uses an excessive amount of alcohol. Finding her drunk, her husband yells at the client about the chronic alcohol abuse. Which reaction should the nurse recognize as the use of the defense mechanism of denial? A. Hiding liquor bottles in a closet B. Yelling at their son for slouching in his chair C. Burning dinner on purpose D. Saying to the spouse, "I don't drink too much!"

ANS: D The nurse should associate the client statement "I don't drink too much!" with the use of the defense mechanism of denial. The client who refuses to acknowledge the existence of a real situation and the feelings associated with it is using the defense mechanism of denial.

A nurse working on an inpatient psychiatric unit is assigned to conduct a 45-minute education group. What should the nurse identify as an appropriate group topic? A. Dream analysis B. Creative cooking C. Paint by number D. Stress management

ANS: D The nurse should identify that teaching clients about stress management is an appropriate education group topic. Nurses should be able to perform the role of client teacher in the psychiatric area. Nurses need to be able to assess a client's learning readiness. Other topics for education groups include medical diagnoses, side effects of medications, and the importance of medication compliance.

The first psychiatric nursing textbook was written by who?

Harriet Bailey

d Battery is an intentional tort in which one individual violates the rights of another through touching without consent. Forcing a patient to take medication after the medication has been refused constitutes battery. The charge of battery can be brought against the nurse.

What is the legal significance of a nurse's action when a patient verbally refuses medication and the nurse gives the medication over the patient's objection? The nurse: a. has been negligent. b. committed malpractice. c. fulfilled the standard of care. d. can be charged with battery.

What is malpractice?

a form of professional negligence, it is the absence of measures to prevent harm to patients and failure to provide reasonable and prudent care

What benchmark was the period of scientific study?

benchmark 2

The first tricyclic anti-depressant, Trofanil, was discovered during which benchmark?

benchmark 3

the patient is assessed, and a plan of care is decided based on the patients need for the following?

safety considerations, intensity of supervision needed, basic needs for shelter and food, ability to function independently, individual and family therapy

____ is the process of isolating a person in a room in which he or she is physically prevented from leaving

seclusion

What is foreseeability?

the reasonable anticipation that harm or injury is likely to result from an act of an omission to act

d

12. The nurse must plan interventions directed toward meeting the client goal: Client will remain safe during hospitalization with the assistance of staff. Which nursing intervention is related to this goal? a. Encourage patient to discuss relationship with family members. b. Assist client to identify three personal weaknesses. c. Observe client for therapeutic effects of psychotropic medication. d. Implement suicide precautions.

b

5. B tells the nurse, "I'm a real freak. I'm a psychiatric patient, in and out of hospitals all the time. None of my friends or relatives is crazy like this." What reply would help B understand the prevalence of mental illness? a. "Comparing yourself with others has no real advantages." b. "Mental illness affects 80% of the adult population in any given year." c. "Nearly 50% of all people age 15 to 55 have had a psychiatric disorder at some time in their lives." d. Unfortunately, there are no answers to your question.

How many of those needing professional help for mental disorders actually seek it?

50%

d

6. The nurse who interviews K notes she is profoundly depressed with thought of wanting to die. Took an unknown amount of pills yesterday. She has refused to do activities of daily living such as bathing. She also has not eaten for the past 5 days, according to her husband. The nurse will code her global assessment of functioning as a. 100 b. 50 c. 25 d. 10

b

7. Maslow offers a theory of human motivation that assumes patients have: a. Developmental tasks and psychosocial crisis b. a hierarchy of needs c. the process of schemata, assimilation and accommodation d. all of the above

a

8. Which statement allows the nurse to suspect that the developmental task of infancy, according to Erickson, was not successfully completed? a. "I'm afraid to allow anyone to really get to know me." b "I'm absolutely right, so don't bother saying more." c "I'm so ashamed because I didn't do it correctly in the first place." d "Andy and I are very close friends."

d

9. T, a 39-year-old businesswoman and single parent of three, is experiencing many feelings of inadequacy in her job and family situation since her 16-year-old daughter ran away several weeks ago. T seeks the help of a therapist specializing in cognitive therapy. The nurse psychotherapist using cognitive therapy will treat T by a. focusing on unconscious mental processes b. negatively reinforcing an undesirable behavior c. discussing ego states d. helping her identify and change faulty thinking

a The DSM-IV-TR gives the criteria used to diagnose each mental disorder. The distracters may not contain diagnostic criteria for a psychiatric illness.

A nurse encounters an unfamiliar psychiatric disorder on a new patient's admission form. To determine criteria used to establish this diagnosis, the nurse should consult which resource? a. Diagnostic and Statistical Manual of Mental Disorders b. A nursing diagnosis handbook c. A psychiatric nursing textbook d. A behavioral health reference manual

An inpatient client, whom the treatment team has determined to be a danger to self, gives notice of intention to leave the hospital. What information should the nurse recognize as having an impact on the treatment team's next action? A. State law determines how long a psychiatric facility can hold a client. B. Federal law determines if the client is competent. C. The client's family involvement will determine if discharge is possible. D. Hospital policies will determine treatment team actions.

ANS: A Most states commonly cite that in an emergency a client who is dangerous to self or others may be involuntarily hospitalized.

A man diagnosed with alcohol dependence experiences his first relapse. During his AA meeting, another group member states, "I relapsed three times, but now have been sober for 15 years." Which of Yalom's curative group factors does this illustrate? A. Imparting of information B. Instillation of hope C. Catharsis D. Universality

ANS: B This scenario is an example of the curative group factor of instillation of hope. This occurs when members observe the progress of others in the group with similar problems and begin to believe that personal problems can also be resolved.

What should be the nurse's primary goal during the preinteraction phase of the nurse-client relationship? A. To evaluate goal attainment and ensure therapeutic closure B. To establish trust and formulate a contract for intervention C. To explore self-perceptions D. To promote client change

ANS: C The nurse's primary goal of the preinteraction phase should be to explore self-perceptions. The nurse should be aware of how any preconceptions may affect his or her ability to care for individual clients. Another goal of the preinteraction phase is to obtain available client information.

c

As Nurse V considers her relationship with K, a client, at what point in the nurse-client relationship should she plan to first address the issue of termination? a. in the working phase b. in the termination phase c. in the orientation phase d. when the client initially brings up the topic

c During the working phase, the nurse strives to assist the patient in making connections among dysfunctional behaviors, thinking, and emotions and offers support while alternative coping behaviors are tried.

During which phase of the nurse-patient relationship can a nurse anticipate that patient issues will be explored and resolved? a. Pre-orientation b. Orientation c. Working d. Termination

c

W is having a series of diagnostic tests. He insists there is nothing wrong with him except a chest cold that he can't "shake off." His wife says he smokes and coughs a lot, has lost 15 pounds, and is easily fatigued. What defense mechanism is W using? a. Regression b. Displacement c. Denial d. Projection

A nurse and patient who developed a therapeutic relationship enter the termination phase. An important nursing intervention for this stage is for the nurse to:discuss feelings about termination with the patient.

Which statement by a patient would the nurse interpret as willingness to collaborate in the nurse-patient relationship? I realize that I have some issues that I need help resolving."

What is psychoanalysis?

a theory of human mental phenomena and behavior focusing on the influence of unconscious forces on the mental state; a method of investigation into the contents of the mind

What is statutory law?

a written law developed from a legislative body, a statute can abolish any rule o common law by specifically stating the rule

Which benchmark was the period of psychotropic drugs?

benchmark 3

Which benchmark was the period of community mental health?

benchmark 4

Homelessness can be directly linked to what benchmark?

benchmark 5

A new patient tells the nurse, "The voices are bothering me." The nurse should first: seek a description of the voices and identify themes.

nurse says, "What step would you like to take next to resolve this issue?" The patient stands up and shouts, "You are so controlling! You want me to do everything your way." What is likely basis of the patient's behavior?

President Kennedy: Joint Commission on Mental Illness and Health was established in what year?

1961

c

31.The wife of a client who has delusions of infidelity asks the nurse if there are any circumstances under which the treatment team may violate the client's right to confidentiality. The specialist must reply that confidentiality may be violated a. under no circumstances b. when questions are asked by law enforcement officers c. if the client threatens the life of another person d. at the discretion of the psychiatrist

c

43. Which of the following is considered an obsession in OCD (obsessive compulsive disorder) a. hand washing b. ritualistic counting c. fear of germs d. excessive order

d The duty to warn a person whose life has been threatened by a psychiatric patient overrides the patient's right to confidentiality. The right to confidentiality is not suspended at the discretion of the therapist or for legal investigations.

A family member of a patient with delusions of persecution asks the nurse, "Are there any circumstances under which the treatment team is justified in violating a patient's right to confidentiality?" The nurse should reply that confidentiality may be breached: a. under no circumstances. b. at the discretion of the psychiatrist. c. when questions are asked by law enforcement. d. if the patient threatens the life of another person.

c The dose of antidepressants for elderly patients is often less than the usual adult dose. The nurse should withhold the medication and consult the health care provider who wrote the order. The nurse's duty is to intervene and protect the patient.

A new antidepressant is prescribed for an elderly patient with major depression, but the dose is more than the usual geriatric dose. The nurse should: a. consult a drug reference. b. teach the patient about possible side effects and adverse effects. c. withhold the medication and confer with the health care provider. d. encourage the patient to increase oral fluids to reduce drug concentration.

d The nurse-patient relationship is structured to provide a model for adaptive interpersonal relationships that can be generalized to others. The distracters apply to theories of cognitive, behavioral, and biological therapy.

A nurse influenced by Peplau's interpersonal theory works with an anxious, withdrawn patient. Interventions should focus on: a. rewarding desired behaviors. b. changing the patient's self-concept. c. administering medications to relieve anxiety. d. enhancing the patient's interactions with others.

A patient is withdrawn and avoids talking to the nurse. The best initial intervention for the nurse would be to: offer to listen and help.

A pt has identified the need for better anger management and tells the nurse, "I'm afraid that someday I might explode. strategy for reducing this patient's fear of losing control is to:talk about these feelings openly and directly.

How is the DSM-IV-TR useful in the practice of psychiatric nursing? (Select all that apply.) A. It considers level of functioning as well as problems. B. It represents progress toward a more holistic view of mind-body. C. It provides a framework for interdisciplinary communication. D. It provides a template for nursing care plans. E. It provides a framework for communication with the client.

ANS: A, B, C The DSM-IV-TR is useful in the practice of psychiatric nursing because it facilitates comprehensive evaluation of the client. It considers the client's current level of functioning, represents a holistic view, and provides a framework for interdisciplinary communication.

Group therapy is strongly encouraged, but not mandatory, on an inpatient psychiatric unit. The unit manager's policy is that clients can make a choice about whether or not to attend group therapy. Which ethical principle does the unit manager's policy preserve? A. Justice B. Autonomy C. Veracity D. Beneficence

ANS: B The unit manager's policy regarding voluntary client participation in group therapy preserves the ethical principle of autonomy. The principle of autonomy presumes that individuals are capable of making independent decisions for themselves and that health-care workers must respect these decisions.

Which is the most significant consequence of the excessive use of defense mechanisms? A. The superego will be suppressed. B. Emotions will be experienced intensely. C. There will be negative psychological and physiological protection. D. Problem solving will be limited.

ANS: D Defense mechanisms become maladaptive when they are used by an individual to such a degree that there is interference with the ability to deal with reality, with effective interpersonal relations, or occupational performance.

President Truman signs the National Mental Health Act in what year?

1946

Hill-Burton Act: Allocated funds for general hospitals to develop psychiatric units in what year?

1947

c

A client tells the nurse, "I don't think I'll ever get out of here." A therapeutic response would be a. Why do you feel that way? b. "Everyone feels that way sometimes." c. "You don't think you're making progress?" d. "Keep up the good work, and you certainly will."

a Milieu therapy is based on the idea that all members of the environment contribute to the planning and functioning of the setting. The distracters are individual therapies that do not fit the description.

A patient would benefit from therapy in which peers as well as staff have a voice in determining patient privileges and psychoeducational topics. Which approach would be best? a. Milieu therapy b. Cognitive therapy c. Short-term dynamic therapy d. Systematic desensitization

A client diagnosed with schizophrenia functions well and is bright, spontaneous, and interactive during hospitalization but then decompensates after discharge. What does the milieu provide that may be missing in the home environment? A. Peer pressure B. Structured programming C. Visitor restrictions D. Mandated activities

ANS: B The milieu, or therapeutic community, provides the client with structured programming that may be missing in the home environment. The therapeutic community provides a structured schedule of activities in which interpersonal interaction and communication with others are emphasized. In the milieu, time is also devoted to personal problems and focus groups.

The nurse-client therapeutic relationship includes which of the following characteristics? (Select all that apply.) A. Meeting the psychological needs of the nurse and the client B. Ensuring therapeutic termination C. Promoting client insight into problematic behavior D. Collaborating to set appropriate goals E. Meeting both the physical and psychological needs of the client

ANS: B, C, D, E The nurse-client therapeutic relationship should include ensuring therapeutic termination, promoting client insight into problematic behavior, collaboration to set appropriate goals, and meeting the physical and psychological needs of the client. The nurse's psychological needs should not be addressed within the nurse-client relationship.

The nurse is interviewing a newly admitted psychiatric client. Which nursing statement is an example of offering a "general lead"? A. "Do you know why you are here?" B. "Are you feeling depressed or anxious?" C. "Yes, I see. Go on." D. "Can you chronologically order the events that led to your admission?"

ANS: C The nurse's statement, "Yes, I see. Go on." is an example of the therapeutic communication technique of a general lead. Offering a general lead encourages the client to continue sharing information.

b The patient has a right to know the conditions of the nurse-patient relationship. If the relationship is to be time limited, the patient should be informed of the number of sessions. If it is open ended, the termination date will not be known at the outset, and the patient should know that the issue will be negotiated at a later date. Termination is usually discussed during the orientation phase. The nurse is responsible for discussing termination early in the relationship.

At what point in the nurse-patient relationship should a nurse first address termination? a. Preorientation phase b. Orientation phase c. Working phase d. Termination phase

What did Tarasoff vs. The regents of the university of California do?

ruled that mental health professionals have a duty to warn of threats of harm to others

What is probable cause?

sufficient credible facts that would induce a reasonably intelligent and prudent person to believe that a cause of action exists

The _____ is the psychoanalytic structure of the mind equivalent to the conscience. It develops in early childhood and provides the ego with an inner control to help cope with the id.

superego

d

10. A therapy that grew out of the need to provide treatment for the posttraumatic stress disorder of returning veterans from WWII and Vietnam is: a. psychoanalysis b. milieu therapy c. systematic desensitization d. short-term dynamic therapy

Mental disorders affect more than ____% of Americans 18 years or older annually

25%

The nurse believes that a patient is having emotional pain. Which remark is most therapeutic? "I hear how painful this is for you. I would like to help you deal with it."

A nurse and patient agree on problems to be addressed during a brief hospital stay. Which inference is correct? The relationship is moving into the working stage.

a The nurse behavior described is typical of overinvolvement. There are no data to support the distracters.

A psychiatric nurse visits one particular patient before work, seeks out the patient during the shift, and spends a few minutes with the patient after going off duty. Which analysis is accurate? The nurse is: a. overinvolved. b. expressing anger. c. experiencing transference. d. looking to be rescued.

A student nurse is learning about the appropriate use of touch when communicating with clients diagnosed with psychiatric disorders. Which statement by the instructor best provides information about this aspect of therapeutic communication? A. "Touch carries a different meaning for different individuals." B. "Touch is often used when deescalating volatile client situations." C. "Touch is used to convey interest and warmth." D. "Touch is best combined with empathy when dealing with anxious clients."

ANS: A Touch can elicit both negative and positive reactions, depending on the people involved and the circumstances of the interaction.

Which nursing statement is a good example of the therapeutic communication technique of focusing? A. "Describe one of the best things that happened to you this week." B. "I'm having a difficult time understanding what you mean." C. "Your counseling session is in 30 minutes. I'll stay with you until then." D. "You mentioned your relationship with your father. Let's discuss that further."

ANS: D This is an example of the therapeutic communication technique of focusing. Focusing takes notice of a single idea or even a single word and works especially well with a client who is moving rapidly from one thought to another.

a Increased levels of GABA reduce anxiety. Acetylcholine and substance P are associated with memory enhancement. Thought disorganization is associated with dopamine. GABA is not associated with sensory perceptual alterations.

The nurse administers a medication that potentiates the action of GABA. Which effect would be expected? a. Reduced anxiety b. Improved memory c. More organized thinking d. Fewer sensory perceptual alterations

b Release of information without patient authorization violates the patient's right to privacy. The other options are acceptable nursing practices.

Which action by a nurse shows a breach of a patient's right to privacy? a. Documents the patient's daily behaviors during hospitalization b. Releases information to the patient's employer without consent c. Discusses the patient's history with other staff during care planning d. Asks family to share information about a patient's prehospitalization behavior

A patient with schizophrenia says to the nurse, "I feel really close to you. You're the only true friend I have." Select the nurse's most therapeutic response.

"Our relationship is professional, but let's explore ways to strengthen friendships in your life."

An inpatient says, "Last time I was here, a primary nurse talked with me every day. This time, different nurses work with me. How can I make progress?" Select the nurse's best response.

"Shift reports, care plans, and progress notes help different nurses work with every patient toward their individual goals."

c

11. Which of the principles of planning nursing intervention to meet client goals is violated in this scenario? Goal: Client will, with the aid of staff, remain safe while in the hospital. Interventions: Allow client to attend off unit programming unsupervised. Client can access all personal belongings. a. The interventions are not nursing focused. b. The interventions are not individualized. c. The interventions are not safe. d. The interventions are not relevant to the goal.

b

13. A nurse behavior that jeopardizes the boundaries of the nurse-client relationship is a. Focusing on the client's needs. b. Allowing the relationship to become social. c. Suspending judgment. d. Recognizing the need for supervision.

c

14. P is being admitted to the psychiatric unit by Nurse G. P was brought to the emergency department after making a suicide attempt by taking an overdose of acetaminophen (Tylenol). P has been lavaged. She appears tense, withdrawn, and frightened. A therapeutic, empathetic response would be that Nurse G tells her: a. "I'd like to sit here with you". b. Tell me more about the suicidal attempt" c. It must be frightening to have just gone through all you have". d. What exactly was going on with you before you tried to hurt yourself".

psychiatric nursing education

1880's: first psychiatric nurse-linda Richards developed nursing care in psychiatric hospitals, directed a school of psychiatric nursing 1920: first psychiatric nursing textbook-harriet bailey 1937: psychiatric nursing included as part of general nursing education curriculum 1952: first psychiatric nursing theorist-Hildegard peplau-developed first mode for psychiatric nursing-wrote interpersonal relations in nursing

The National Institute of Mental Health was established in what year?

1949

What year did the Community Mental Health Centers Act pass?

1963

d A high level of valuing is acting on one's belief. Autonomy is evident when the nurse helps patient weigh alternatives and their consequences before the patient makes a decision. Autonomy or self-determination is not the issue in any of the other behaviors.

21. Which behavior shows that a nurse values autonomy? The nurse: a. sets limits on a patient's romantic overtures toward the nurse. b. suggests one-on-one supervision for a patient who is suicidal. c. informs a patient that the spouse will not be in during visiting hours. d. discusses alternatives and helps the patient weigh the consequences.

Axis __ is general medical conditions that relate to axes 1 or 2 or have a bearing on treatment such as neoplasms or endocrine disorders

3

Axis __ is global assessment of functioning, on a scale of 0-100 a score of 30 means that the patient's behavior is highly influenced by delusions and hallucinations

5

b This behavior is typical of a child around the age of 2 years whose developmental task is to develop autonomy. The distracters indicate the child's behavior is abnormal.

A 2-year-old child often displays negative behaviors. The parent says, "My child refuses toilet training and shouts 'No!' when given directions. What do you think is wrong?" Select the nurse's best reply. a. "The child needs firmer control. It is important to set limits now." b. "This is normal for your child's age. The child is striving for independence." c. "There may be developmental problems. Most children are toilet trained by age 2." d. "Some undesirable attitudes are developing. A child psychologist can help you develop a plan."

b The anal stage occurs from age 1 to 3 years and has as its focus toilet training and learning to delay immediate gratification. The oral stage occurs between birth and 1 year. The phallic stage occurs between 3 and 5 years, and the genital stage occurs between age 13 and 20 years.

A 26-month-old displays negative behavior, refuses toilet training, and often says, "No!" Which stage of psychosexual development is evident? a. Oral b. Anal c. Phallic d. Genital

a The id operates on the pleasure principle, seeking immediate gratification of impulses. The ego acts as a mediator of behavior and weighs the consequences of the action, perhaps determining that taking the toy is not worth the mother's wrath. The superego would oppose the impulsive behavior as "not nice." The preconscious is a level of awareness.

A 4-year-old grabs toys from siblings and says, "I want that now!" The siblings cry, and the child's parent becomes upset with the behavior. Using Freudian theory, the nurse can interpret this behavior as a product of impulses originating in the: a. id. b. ego. c. superego. d. preconscious.

a The patient sees self as needing multiple explanations of new tasks at work and allows the parents to make decisions, even though she is 40 years old. These behaviors indicate a poorly developed self-concept.

A 40-year-old who lives with parents and works at an unchallenging job says, "I'm as happy as anyone else, even though I don't socialize much outside of work. My work is routine, but when new things come up, my boss explains things a few times to make sure I catch on. At home, my parents make decisions for me, and I go along with their ideas." The nurse should identify interventions to improve this patient's: a. self-concept. b. overall happiness. c. appraisal of reality. d. control over behavior.

a

A client is noted to have a high level of non-goal-directed motor activity, running from chair to chair in the solarium. He is wide-eyed and seems terror-stricken. He cries, "They're coming! They're coming!" He neither follows staff direction nor responds to verbal efforts to calm him. A nursing diagnosis of high priority is a. Risk for injury b. Self-care deficit c. Disturbed energy field d. Impaired verbal communication

d Cognitive theory suggests that thought processes are the basis of emotions and behavior. Changing faulty learning makes development of new adaptive behaviors possible. The distracters relate to psychoanalytic therapy, biological therapy, and aversion therapy.

A cognitive strategy the nurse could use to help a dependent patient would be: a. avoidance training. b. filling the patient's pill minder. c. interpreting the patient's dream content. d. examining the patient's fears related to being independent.

a After termination of a long-term relationship, the patient and new nurse usually have to begin at ground zero, the orientation phase, to build a new relationship. If termination is successfully completed, the orientation phase sometimes progresses quickly to the working phase. Other times, even after successful termination, the orientation phase may be prolonged.

A community mental health nurse has worked with a patient for 3 years but is moving and must terminate the relationship. When a new nurse begins work with this patient, what is the starting point for the relationship? a. Begin at the orientation phase b. Continue the working relationship c. Start with informal social interactions d. Return to the emotional catharsis phase

a An advocate defends or asserts another's cause, particularly when the other person lacks the ability to do that for self. Examples of individual advocacy include helping patients understand their rights or make decisions. On a community scale, advocacy includes political activity, public speaking, and publication in the interest of improving the human condition. Since funding is necessary to deliver quality programming for persons with mental illness, the letter-writing campaign advocates for that cause on behalf of patients who are unable to articulate their own needs.

A new bill introduced in Congress would reduce funding for care of persons with mental illness. Groups of nurses write letters to their elected representatives in opposition to the legislation. Which role have the nurses fulfilled? a. Advocacy b. Attending c. Recovery d. Evidence-based practice

d The admitting physician would use axis IV to record psychosocial and environmental problems pertinent to the patient's situation, providing another source of information for the nurse. Persistent questioning may cause the patient to withdraw. The other distracters demonstrate violation of the patient's privacy rights and are not an effective solution.

A newly admitted patient is uncommunicative about recent life events. The nurse suspects marital and economic problems, but the social worker's assessment is not yet available. Select the nurse's best action. a. Focus assessment questions on these two topics. b. Ask another patient who shares a room with this patient. c. Avoid seeking information on these topics at this time. d. Refer to axis IV of the DSM-IV-TR in the medical record.

c Countertransference is the nurse's response to a patient that is based on the nurse's unconscious needs, conflicts, problems, or view of the world.

A nurse assesses an elderly patient who was found wandering and confused. The nurse feels sad and reflects, "She's like my grandmother...so helpless." Which term best applies to the nurse's response? a. Rapport b. Transference c. Countertransference d. Defensive coping reaction

d Norepinephrine is the neurotransmitter associated with sympathetic nervous system stimulation, preparing the individual for "fight or flight." GABA is a mediator of anxiety level. A high concentration of histamine is associated with an inflammatory response. A high concentration of acetylcholine is associated with parasympathetic nervous system stimulation.

A nurse assesses that a patient has fear as well as increased heart rate and blood pressure. The nurse suspects increased activity of which neurotransmitter? a. GABA b. Histamine c. Acetylcholine. d. Norepinephrine

b Patients with mental illness retain their civil rights unless there is clear, cogent, and convincing evidence of dangerousness. The patient in this situation presents no evidence of dangerousness. The nurse, an as advocate and educator, should seek more information about the patient's decision and should not force the medication.

A nurse at the mental health center prepares to administer a scheduled injection of haloperidol decanoate (Haldol depot) to a patient with schizophrenia. As the nurse swabs the site, the patient shouts, "Stop, stop. I don't want to take that medicine anymore because I hate the side effects." Select the nurse's first action. a. Assemble other staff for a show of force and proceed with the injection, using restraint if necessary. b. Stop the medication administration procedure and say to the patient, "Tell me more about the side effects you've been having." c. Proceed with the injection but explain to the patient that there are medications that will help reduce the unpleasant side effects. d. Say to the patient, "Since I've already drawn the medication in the syringe, I'm required to give it, but let's talk to the doctor about delaying next month's dose."

a When the role of the nurse and the role of the patient shift, boundary blurring may arise. In this situation, the nurse is becoming overinvolved with the patient as a probable result of unrecognized countertransference. When boundary issues occur, the need for supervision exists. The situation does not describe sexual harassment. Data are not present to suggest positive regard or transference.

A nurse believes, "I'm the only one who truly understands this patient. Other staff members are too critical." Which situation is evident? a. Boundary blurring b. Sexual harassment c. Positive regard d. Transference

d The nurse is accountable for the relationship. Objectivity is threatened by strong positive or negative feelings toward a patient. Supervision is necessary to work through countertransference feelings.

A nurse caring for a withdrawn, suspicious patient recognizes development of feelings of anger toward the patient. The nurse should: a. suppress the angry feelings. b. explore the anger openly and directly with the patient. c. ask the nurse manager to assign the patient to another nurse. d. discuss the anger with a clinician during a supervisory session.

d Increased acetylcholine plays a role in learning and memory. Preventing destruction of acetylcholine by acetylcholinesterase would result in higher levels of acetylcholine, with the potential for improved memory. GABA affects anxiety rather than memory. Increased dopamine would cause symptoms associated with schizophrenia or mania rather than improve memory. Decreasing dopamine at receptor sites is associated with Parkinson's disease rather than improving memory.

A nurse could anticipate that treatment for a patient with memory difficulties might include medications designed to: a. inhibit GABA. b. increase dopamine activity. c. reduce neurotensin metabolism. d. prevent destruction of acetylcholine.

c The use of five axes requires assessment beyond diagnosis of a mental disorder and includes relevant medical conditions, psychosocial and environmental problems, and global assessment of functioning. The DSM-IV-TR does not include treatment plans or nursing diagnoses. It does not use specific biopsychosocial theories.

A nurse explains the multiaxial DSM-IV-TR to a psychiatric technician and includes information that it: a. focuses on plans for treatment. b. includes nursing and medical diagnoses. c. classifies problems in multiple areas of functioning. d. uses the framework of a specific biopsychosocial theory.

a The distracters describe events that occur in social or intimate relationships. Only the correct answer describes elements of a therapeutic relationship.

A nurse explains to the family of a mentally ill patient how the nurse-patient relationship differs from other interpersonal relationships. Which is the best explanation? "The focus: a. is on the patient. Problems are discussed by the nurse and patient, but solutions are implemented by the patient." b. shifts from nurse to patient as the relationship develops. Advice is given by both and solutions implemented." c. is creation of a partnership in which each member is concerned with growth and satisfaction of the other." d. of the relationship is socialization. Mutual needs are met and feelings are shared openly."

b Clinical epidemiology is a broad field that addresses what happens to people with illnesses seen by providers of clinical care. This study is concerned with the effectiveness of various interventions. Prevalence refers to numbers of new cases. Descriptive epidemiology provides estimates of the rates of disorders in a general population and its subgroups. Experimental epidemiology tests presumed assumptions between a risk factor and a disorder.

A nurse is part of a multidisciplinary team working with groups of depressed patients. Half the patients receive supportive interventions and antidepressant medication. The other half receives only medication. The team measures outcomes for each group. Which type of study is evident? a. Prevalence b. Clinical epidemiology c. Descriptive epidemiology d. Experimental epidemiology

b When a specific duration for sessions has been set, the nurse must adhere to the schedule. Leaving the first patient would be abandonment and may destroy trust the patient had in the nurse. Adhering to the contract demonstrates that the nurse can be trusted and that the patient and the sessions are important. The distracters violate terms of the contract.

A nurse is talking with a patient, and 5 minutes remain in the session. This patient has been silent most of the session. Another patient interrupts and says to the nurse, "I really need to talk to you." The nurse should: a. invite the interrupting patient to join the session with the current patient. b. tell the interrupting patient, "This session is 5 more minutes, then I will talk with you." c. say to the interrupting patient, "I am not available to talk with you at the present time." d. end the unproductive session with the current patient. Spend time with the next patient.

d Both retirees are in middle adulthood, when the developmental crisis to be resolved is generativity versus self-absorption. One exemplifies generativity; the other embodies self-absorption. This developmental crisis would show a contrast between relating to others in a trusting fashion or being suspicious and lacking trust. Failure to negotiate this developmental crisis would result in a sense of inferiority or difficulty learning and working as opposed to the ability to work competently. Behaviors that would be contrasted would be emotional isolation and the ability to love and commit oneself.

A nurse listens to a group of recent retirees. One says, "I volunteer with Meals on Wheels, coach teen sports, and do church visitation." Another laughs and says, "I'm too busy taking care of myself to volunteer to help others." Which developmental task do these statements contrast? a. Trust and mistrust b. Intimacy and isolation c. Industry and inferiority d. Generativity and self-absorption

d The prevalence for Alzheimer's disease is 10% for persons older than 65 and 50% for persons older than 85. The prevalence of schizophrenia is 1.1% per year. The prevalence of bipolar disorder is 2.6%. It is important for the nurse to provide information rather than probe the reason for the person's question.

A nurse participating in a community health fair is asked, "What is the most prevalent mental disorder in the United States?" Select the nurse's best response. a. Schizophrenia b. "Why do you ask?" c. Bipolar disorder d. Alzheimer's disease

d Accepting a gift creates a social rather than therapeutic relationship with the patient and blurs the boundaries of the relationship. A caring nurse will acknowledge the patient's gesture of appreciation, but this gift should not be accepted.

A nurse provided psychiatric home care services to a patient for 6 months, but now the patient will begin a psychosocial rehabilitation program. On the nurse's final home visit, the patient gives the nurse a gold angel pin and says, "Thank you for being my guardian angel when I needed help." Select the nurse's best response. a. "I'm happy you have made so much improvement. Thank you for the pin." b. "Our agency's policies and procedures prohibit me from accepting your gift." c. "All nurses care. It's rewarding when patients recognize how hard we work." d. "I'm glad you've made progress and that I helped, but I cannot accept the gift."

A patient is hospitalized for severe depression. Knowing that the patient will be discharged after a short stay, what is the nurse's first priority? Develop a goal-directed, problem-centered relationship.

A nurse tells a patient, "I know how you feel. My spouse can be very insensitive too. I am also considering divorce." Analysis suggests that the nurse is: self-disclosing inappropriately.

a The need for food and hygiene are physiological and therefore take priority over psychological or meta-needs in care planning.

A nurse uses Maslow's hierarchy of needs to plan care for a patient with mental illness. Which problem will receive priority? The patient: a. refuses to eat or bathe. b. reports feelings of alienation from family. c. is reluctant to participate in unit social activities. d. is unaware of medication action and side effects.

d Genuineness is a desirable characteristic involving awareness of one's own feelings as they arise and the ability to communicate them when appropriate. The other possible options are undesirable in a therapeutic relationship.

A nurse wants to demonstrate genuineness with a patient diagnosed with schizophrenia. The nurse should: a. use extensive self-disclosure in patient interactions. b. encourage dependence on the nurse for support and reassurance. c. consistently make interpretive judgments about the patient's behavior. d. be aware of own feelings and use congruent communication strategies.

d The patient asked for information, and the correct response is most accurate. Neurotransmitters are chemical substances that function as messengers in the central nervous system. They are released from the axon terminal, diffuse across the synapse, and attach to specialized receptors on the postsynaptic neuron. The distracters either do not answer the patient's question or provide untrue, misleading information.

A patient asks, "What are neurotransmitters? The doctor said mine are imbalanced." Select the nurse's best response. a. "How do you feel about having imbalanced neurotransmitters?" b. "You must feel relieved to know that your problem has a physical basis." c. "Neurotransmitters are substances we eat daily that influence memory and mood." d. "Neurotransmitters are natural chemicals that pass messages between brain cells."

d Fixation at the oral stage sometimes produces dependent infantile behaviors in adults. Latency fixations often result in difficulty identifying with others and developing social skills, resulting in a sense of inadequacy and inferiority. Phallic fixations result in having difficulty with authority figures and poor sexual identity. Anal fixation sometimes results in retentiveness, rigidity, messiness, destructiveness, and cruelty.

A patient expresses the desire to be cared for by others and often behaves in a helpless fashion. The patient's needs relate to which stage of psychosexual development? a. Latency b. Phallic c. Anal d. Oral

d Patients often unconsciously use testing behaviors to determine whether the nurse is able to set limits or will abandon them if they behave in an unlikable way.

A patient frequently asks the nurse for extra snacks, implying more willingness to talk if these items are provided. The nurse should assess this behavior as: a. typical of transference reactions. b. indicative of feelings of insecurity. c. reflecting resistance to involvement. d. testing the nurse's clinical competence.

c The situation describes psychodynamic psychotherapy. The distracters use other techniques.

A patient had psychotherapy weekly for 5 months. The therapist used free association, dream analysis, and facilitated transference to help the patient understand conflicts and foster change. Select the term that applies to this method. a. Rational-emotive behavior therapy b. Cognitive-behavioral therapy c. Psychodynamic psychotherapy d. Operant conditioning

a Laws regarding child abuse reporting discovered by a professional during the suspected abuser's alcohol or drug treatment differ by state. Federal law supersedes state law and prohibits disclosure without a court order except in instances in which the report can be made anonymously or without identifying the abuser as a patient in an alcohol or drug treatment facility.

A patient in alcohol rehabilitation reveals to the nurse, "I feel terrible guilt for sexually abusing my 6-year-old before I was admitted." Select the nurse's most important action. a. Anonymously report the abuse by phone to the local child protection agency. b. Reply, "I'm glad you feel comfortable talking to me about it." c. Respect nurse-patient relationship confidentiality. d. File a written report on agency letterhead.

a The behaviors in the stem develop as the result of attitudes formed during the oral stage, when an infant first learns to relate to the environment. Anal-stage traits include stinginess, stubbornness, orderliness, or their opposites. Phallic stage traits include flirtatiousness, pride, vanity, difficulty with authority figures, and difficulties with sexual identity. Genital stage traits include the ability to form satisfying sexual and emotional relationships with members of the opposite sex, emancipation from parents, a strong sense of personal identity, or the opposites of these traits.

A patient is suspicious and frequently manipulates others. To which psychosexual stage do these traits relate? a. Oral b. Anal c. Phallic d. Genital

c These statements show severe self-doubt, indicating that the crisis of gaining control over the environment was not successfully met. Unsuccessful resolution of the crisis of initiative versus guilt results in feelings of guilt. Unsuccessful resolution of the crisis of trust versus mistrust results in poor interpersonal relationships and suspicion of others. Unsuccessful resolution of the crisis of generativity versus self-absorption results in self-absorption that limits the ability to grow as a person.

A patient says, "I never know the answers," and "My opinion doesn't count." The nurse correctly assesses that this patient had difficulty resolving which psychosocial crisis? a. Initiative versus guilt b. Trust versus mistrust c. Autonomy versus shame and doubt d. Generativity versus self-absorption

b Nurses should encourage patients to work at their optimal level of functioning. The nurse does not act for the patient unless necessary. Acting for a patient increases feelings of helplessness and dependency.

A patient says, "I'm still on restriction, but I want to attend some off-unit activities. Would you ask the doctor to change my privileges?" Select the nurse's best response. a. "I'll be glad to mention it when I see the doctor today." b. "That's a good topic for you to take up with your doctor." c. "Why are you asking me when you're able to speak for yourself?" d. "I'm glad you feel comfortable asking me to help you with that request."

c Values guide beliefs and actions. The individuals stating their positions place different values on life and autonomy. Nurses must be aware of their own values and be sensitive to the values of others.

A patient says, "People should be allowed to commit suicide without interference from others." A nurse replies, "You're wrong. Nothing is bad enough to justify death." What is the best analysis of this interchange? a. The patient is correct. b. Neither person is correct. c. The statements reflect differing values. d. The nurse has responded therapeutically.

a Being overly helpful is a classic example of role boundary blurring. The nurse's response does not reflect the modeling of healthy behavior. There is no data to indicate the patient has manipulated the nurse.

A patient who recently attempted suicide talks with the nurse about wanting to take a walk on hospital grounds. The nurse responds by telling the patient, "I will talk with the psychiatrist on your behalf." Select the accurate analysis of this interaction. a. The nurse is behaving in an overly helpful way. b. The nurse is showing positive regard for the patient. c. The nurse is modeling healthy behaviors for the patient. d. The patient has manipulated the nurse into taking this action.

b The symptoms describe mania, which is effectively treated by mood stabilizers such as lithium and selected anticonvulsants (carbamazepine, valproic acid, and lamotrigine). Drugs from the other classifications listed are not effective in the treatment of mania.

A patient with bipolar disorder has an unstable mood, aggressiveness, agitation, talkativeness, and irritability. The nurse begins care planning based on the expectation that the health care provider is most likely to prescribe a medication classified as a(n): a. anticholinergic. b. mood stabilizer. c. psychostimulant. d. antidepressant.

a Patients have the right to be treated with dignity and respect. Patients should never be made the object of jokes about their illness. Patient emotional abuse has been demonstrated, not negligence. This example is not clearly defamation. Patient abuse, not supervisory liability, is the issue.

A patient with psychosis asks a psychiatric technician, "What's the matter with me?" The technician replies, "Your wing nuts need tightening." The nurse who overheard the exchange will need to take action based on: a. violation of the patient's right to be treated with dignity and respect. b. the nurse's obligation to report caregiver negligence. c. preventing defamation of the patient's character. d. supervisory liability.

d Documentation must be specific and detail the key aspects of care. It should demonstrate implementation of the least restrictive alternative. Justification for why a patient was secluded for 8 hours should be recorded, along with interventions attempted in an effort to avoid seclusion. Documentation should include a description of behavior and verbalizations, interventions tried and their outcomes, and the name of the health care provider ordering the use of seclusion.

A patient with psychosis became aggressive, struck another patient, and required seclusion. Select the best documentation. a. Patient struck another patient who attempted to leave day room to go to bathroom. Seclusion necessary at 1415. Plan: Maintain seclusion for 8 hours and keep these two patients away from each other for 24 hours. b. Seclusion ordered by physician at 1415 after command hallucinations told the patient to hit another patient. Careful monitoring of patient maintained during period of seclusion. c. Seclusion ordered by MD for aggressive behavior. Begun at 1415. Maintained for 2 hours without incident. Outcome: Patient calmer and apologized for outburst. d. Patient pacing, shouting. Haloperidol 5 mg given PO at 1300. No effect by 1315. At 1415 patient yelled, "I'll punch anyone who gets near me," and struck another patient with fist. Physically placed in seclusion at 1420. Seclusion order obtained from MD at 1430.

d The information given centers on relationships with others, which are described as intense and unstable. The relationships of mentally healthy individuals are stable, satisfying, and socially integrated. Data are not present to describe work effectiveness, communication skills, or activities.

A patient's relationships are intense and unstable. The patient initially idealizes the significant other and then devalues them, resulting in frequent feelings of emptiness. This patient will benefit from interventions to develop which aspect of mental health? a. Effectiveness in work b. Communication skills c. Productive activities d. Fulfilling relationships

a The community is a less restrictive alternative than hospitals for treatment of persons with mental illness. The distracters are incorrect and part of the stigma of mental illness.

A person in the community asks, "Why aren't people with mental illness kept in state institutions anymore?" Select the nurse's best response. a. "Less restrictive settings are available now to care for individuals with mental illness." b. "There are fewer persons with mental illness, so less hospital beds are needed." c. "Most people with mental illness are still in psychiatric institutions." d. "Psychiatric institutions violated patients' rights."

c The structure of the therapeutic environment has as foci an accepting atmosphere and provision of opportunities for practicing interpersonal skills. Both constructs are directly attributable to Sullivan's theory of interpersonal relationships. Sullivan's interpersonal theory did not specifically consider use of restraint or seclusion. Assessment based on developmental level is more the result of Erikson's theories. Sequencing nursing actions based on patient priority needs is related to Maslow's hierarchy of needs.

A psychiatric technician says, "Common sense is the most important part of working with people who have mental illness. Theories are just something to fill up textbooks." The nurse wants to educate the technician by identifying which common use of Sullivan's theory? a. The method nurses use to determine the best sequence for nursing actions b. The ongoing use of restraint and seclusion as behavior-management tools c. The structure of the therapeutic milieu of most behavioral health units d. Assessment tools based on age-appropriate versus arrested behaviors

c Positive or negative feelings of the patient toward the therapist indicate transference. Transference is a psychoanalytic concept that can be used to explore previously unresolved conflicts. The distracters are more related to biological therapy and supportive psychotherapy. Use of psychoeducational materials is a common "homework" assignment used in cognitive therapy.

A psychotherapist works with an anxious, dependent patient. Which strategy is most consistent with psychoanalytic psychotherapy? a. Identifying the patient's strengths and assets b. Praising the patient for describing feelings of isolation c. Focusing on feelings developed by the patient toward the therapist d. Providing psychoeducation and emphasizing medication adherence

d Cognitive therapy emphasizes the importance of changing erroneous ways people think about themselves. Once faulty thinking is changed, the individual's behavior changes. The distracters describe a psychoanalytic approach and behavior modification.

A single parent who is employed full time complains of feelings of inadequacy related to work and family. The parent seeks help from a therapist who specializes in cognitive behavioral therapy. The therapist will treat the parent by: a. discussing ego states the parent experiences. b. negatively reinforcing undesirable behaviors. c. promoting assertive behavior at home and work. d. helping the parent identify and change faulty thinking.

b Sullivan believed that the nurse's role includes educating patients and assisting them in developing effective interpersonal relationships. Mutuality, respect for the patient, unconditional acceptance, and empathy are cornerstones of Sullivan's theory. The nurse who does not interact with the patient cannot demonstrate these cornerstones. Observations provide only objective data. Priority nursing diagnoses usually cannot be accurately established without subjective data from the patient. The other distracters relate to Maslow and behavioral theory.

A student nurse says, "I don't need to interact with my patients. I learn what I need to know by observation." An instructor can best interpret the nursing implications of Sullivan's theory to this student by responding: a. "Interactions are required in order to help you develop therapeutic communication skills." b. "Nurses cannot be isolated. We must interact to provide patients with opportunities to practice interpersonal skills." c. "Observing patient interactions will help you formulate priority nursing diagnoses and appropriate interventions." d. "It is important to note patients' behavioral changes, because these signify adjustments in personality."

c A voluntarily admitted patient has the right to demand and obtain release in most states. However, as a patient advocate, the nurse is responsible for weighing factors related to the patient's wishes and best interests. By asking for information, the nurse may be able to help the patient reconsider the decision. Facilitating discharge without consent is not in the patient's best interests before exploring the reason for the request.

A voluntarily hospitalized patient tells the nurse, "Get me the forms for discharge. I want to leave now." Select the nurse's best response. a. "I will get the forms for you right now and bring them to your room." b. "Since you signed your consent for treatment, you may leave if you desire." c. "I will get them for you, but let's talk about your decision to leave treatment." d. "I cannot give you those forms without your health care provider's knowledge."

A client is questioning the nurse about a newly prescribed medication, acamprosate calcium (Campral). Which is the most appropriate reply by the nurse? A. "This medication will help you maintain your abstinence." B. "This medication will cause uncomfortable symptoms if you combine it with alcohol." C. "This medication will decrease the effect alcohol has on your body." D. "This medication will lower your risk of experiencing a complicated withdrawal."

ANS: A Campral has been approved by the U.S. Food and Drug Administration (FDA) for the maintenance of abstinence from alcohol in clients diagnosed with alcohol dependence who are abstinent at treatment initiation.

A client diagnosed with schizophrenia receives fluphenazine decanoate (Prolixin Decanoate) from a home health nurse. The client refuses medication at one regularly scheduled home visit. Which nursing intervention is ethically appropriate? A. Allow the client to decline the medication and document. B. Tell the client that if the medication is refused, hospitalization will occur. C. Arrange with a relative to add medication to the client's morning orange juice. D. Call for help to hold the client down while the injection is administered.

ANS: A It is ethically appropriate for the nurse to allow the client to decline the medication and provide accurate documentation. The client's right to refuse treatment should be upheld unless the refusal puts the client or others in harm's way.

A nurse is assessing a client diagnosed with schizophrenia for the presence of hallucinations. Which therapeutic communication technique used by the nurse is an example of making observations? A. "You appear to be talking to someone I do not see." B. "Please describe what you are seeing." C. "Why do you continually look in the corner of this room?" D. "If you hum a tune, the voices may not be so distracting."

ANS: A The nurse is making an observation when stating, "You appear to be talking to someone I do not see." Making observations involves verbalizing what is observed or perceived. This encourages the client to recognize specific behaviors and make comparisons with the nurse's perceptions.

A client has a history of daily bourbon drinking for the past 6 months. He is brought to an emergency department by family who report that his last drink was 1 hour ago. It is now 12 midnight. When should a nurse expect this client to exhibit withdrawal symptoms? A. Between 3 a.m. and 11 a.m. B. Shortly after a 24-hour period C. At the beginning of the third day D. Withdrawal is individualized and cannot be predicted.

ANS: A The nurse should expect that this client will begin experiencing withdrawal symptoms from alcohol between 3 a.m. and 11 a.m. Symptoms of alcohol withdrawal usually occur within 4 to 12 hours of cessation or reduction in heavy and prolonged alcohol use.

A nurse is assessing a set of 15-year-old identical twins who respond very differently to stress. One twin becomes anxious and irritable, while the other withdraws and cries. How should the nurse explain these different responses to stress to the parents? A. Reactions to stress are relative rather than absolute; individual responses to stress vary. B. It is abnormal for identical twins to react differently to similar stressors. C. Identical twins should share the same temperament and respond similarly to stress. D. Environmental influences to stress weigh more heavily than genetic influences.

ANS: A The nurse should explain to the parents that although the twins have identical DNA, there are several other factors that affect reactions to stress. Mental health is a state of being that is relative to the individual client. Environmental influences and temperament can affect stress reactions.

During an inpatient educational group, a client shouts out, "This information is worthless. Nothing you have said can help me." These statements indicate to the nurse leader that the client is assuming which group role? A. The group role of aggressor B. The group role of initiator C. The group role of gatekeeper D. The group role of blocker

ANS: A The nurse should identify that the client is assuming the group role of the aggressor. The aggressor expresses negativism and hostility toward others in the group or to the group leader and may use sarcasm in an effort to degrade the status of others.

A client requests information on several medications in order to make an informed choice about management of depression. A nurse should provide this information to facilitate which ethical principle? A. Autonomy B. Beneficence C. Nonmaleficence D. Justice

ANS: A The nurse should provide the information to support the client's autonomy. A client who is capable of making independent choices should be permitted to do so. In instances when clients are incapable of making informed decisions, a legal guardian or representative would be asked to give consent.

During a therapeutic group, two clients engage in an angry verbal exchange. The nurse leader interrupts the exchange and excuses both of the clients from the group. The nurse has demonstrated which leadership style? A. Autocratic B. Democratic C. Laissez-faire D. Bureaucratic

ANS: A The nurse who excuses clients from the group has demonstrated an autocratic leadership style. An autocratic leadership style may be useful in certain situations that require structure and limit setting. Democratic leaders focus on the members of the group and group-selected goals. Laissez-faire leaders provide no direction to group members.

A client has made the decision to leave her alcoholic husband. She is feeling very depressed. Which nontherapeutic statement by the nurse conveys sympathy? A. "You are feeling very depressed. I felt the same way when I decided to leave my husband." B. "I can understand you are feeling depressed. It was a difficult decision. I'll sit with you." C. "You seem depressed. It was a difficult decision to make. Would you like to talk about it?" D. "I know this is a difficult time for you. Would you like a prn medication for anxiety?"

ANS: A The nurse's statement, "You are feeling very depressed. I felt the same when I decided to leave my husband," is a nontherapeutic statement that conveys sympathy. Sympathy implies that the nurse shares what the client is feeling and by this personal expression alleviates the nurse's distress

What should be the priority nursing diagnosis for a client experiencing alcohol withdrawal? A. Risk for injury R/T central nervous system stimulation B. Disturbed thought processes R/T tactile hallucinations C. Ineffective coping R/T powerlessness over alcohol use D. Ineffective denial R/T continued alcohol use despite negative consequences

ANS: A The priority nursing diagnosis for a client experiencing alcohol withdrawal should be risk for injury R/T central nervous system stimulation. Alcohol withdrawal may include the following symptoms: course tremors of hands, tongue, or eyelids; seizures; nausea or vomiting; malaise or weakness; tachycardia; sweating; elevated blood pressure; anxiety; depressed mood; hallucinations; headache; and insomnia.

A client is struggling to explore and solve a problem. Which nursing statement would verbalize the implication of the client's actions? A. "You seem to be motivated to change your behavior." B. "How will these changes affect your family relationships?" C. "Why don't you make a list of the behaviors you need to change." D. "The team recommends that you make only one behavioral change at a time."

ANS: A This is an example of the therapeutic communication technique of verbalizing the implied. Verbalizing the implied puts into words what the client has only implied or said indirectly.

During a nurse-client interaction, which nursing statement may belittle the client's feelings and concerns? A. "Don't worry. Everything will be alright." B. "You appear uptight." C. "I notice you have bitten your nails to the quick." D. "You are jumping to conclusions."

ANS: A This nursing statement is an example of the nontherapeutic communication block of belittling feelings. Belittling feelings occur when the nurse misjudges the degree of the client's discomfort, thus a lack of empathy and understanding may be conveyed.

Which nursing response is an example of the nontherapeutic communication block of requesting an explanation? A. "Can you tell me why you said that?" B. "Keep your chin up. I'll explain the procedure to you." C. "There is always an explanation for both good and bad behaviors." D. "Are you not understanding the explanation I provided?"

ANS: A This nursing statement is an example of the nontherapeutic communication block of requesting an explanation. Requesting an explanation is when the client is asked to provide the reason for thoughts, feelings, behaviors, and events. Asking "why" a client did something or feels a certain way can be very intimidating and implies that the client must defend his or her behavior or feelings.

Which client statement may indicate a transference reaction? A. "I need a real nurse. You are young enough to be my daughter and I don't want to tell you about my personal life." B. "I deserve more that I am getting here. Do you know who I am and what I do? Let me talk to your supervisor." C. "I don't seem to be able to relate to people. I would rather stay in my room and be by myself." D. "My mother is the source of my problems. She has always told me what to do and what to say."

ANS: A Transference occurs when a client unconscientiously displaces, or "transfers," to the nurse feelings formed toward a person from the past.

A client is admitted for alcohol detoxification. During detoxification, which symptoms should the nurse expect to assess? A. Gross tremors, delirium, hyperactivity, and hypertension B. Disorientation, peripheral neuropathy, and hypotension C. Oculogyric crisis, amnesia, ataxia, and hypertension D. Hallucinations, fine tremors, confabulation, and orthostatic hypotension

ANS: A Withdrawal is defined as the physiological and mental readjustment that accompanies the discontinuation of an addictive substance. Symptoms can include gross tremors, delirium, hyperactivity, hypertension, nausea, vomiting, tachycardia, hallucinations, and seizures.

During a group discussion, members freely interact with each other. Which member statement is an example of Yalom's curative group factor of imparting information? A. "I found a Web site explaining the different types of brain tumors and their treatment." B. "My brother also had a brain tumor and now is completely cured." C. "I understand your fear and will be by your side during this time." D. "My mother was also diagnosed with cancer of the brain."

ANS: A Yalom's curative group factor of imparting information involves sharing knowledge gained through formal instruction as well as by the sharing of advice and suggestions by other group members.

A nurse is assessing a client who appears to be experiencing mild anxiety during questioning. Which symptoms might the client demonstrate? (Select all that apply.) A. Fidgeting B. Laughing inappropriately C. Palpitations D. Nail biting E. Limited attention span

ANS: A, B, D The nurse should assess that fidgeting, laughing inappropriately, and nail biting are indicative of heightened stress levels. The client would not be diagnosed with mental illness unless there is significant impairment in other areas of daily functioning. Other indicators of more serious anxiety are restlessness, difficulty concentrating, muscle tension, and sleep disturbance.

A nursing instructor is teaching students about cirrhosis of the liver. Which of the following student statements about the complications of hepatic encephalopathy should indicate that further student teaching is needed? (Select all that apply.) A. "A diet rich in protein will promote hepatic healing." B. "This condition leads to a rise in serum ammonia resulting in impaired mental functioning." C. "In this condition, blood accumulates in the abdominal cavity." D. "Neomycin and lactulose are used in the treatment of this condition." E. "This condition is caused by the inability of the liver to convert ammonia to urea."

ANS: A, C The nursing instructor should understand that further teaching is needed if the nursing student states that a diet rich in protein will promote hepatic healing and that this condition causes blood to accumulate in the abdominal cavity (ascites), because these are incorrect statements. The treatment of hepatic encephalopathy requires abstention from alcohol, temporary elimination of protein from the diet, and reduction of intestinal ammonia using neomycin or lactulose. This condition occurs in response to the inability of the liver to convert ammonia to urea for excretion.

Which of the following statements should a nurse recognize as true about defense mechanisms? (Select all that apply.) A. They are employed when there is a threat to biological or psychological integrity. B. They are controlled by the id and deal with primal urges. C. They are used in an effort to relieve mild to moderate anxiety. D. They are protective devices for the superego. E. They are mechanisms that are characteristically self-deceptive.

ANS: A, C, E Defense mechanisms are employed by the ego in the face of threats to biological and psychological integrity, in an effort to relieve mild to moderate anxiety. Because they redirect focus, they are characteristically self-deceptive.

After disturbing the peace, an aggressive, disoriented, unkempt, homeless individual is escorted to an emergency department by police. The client threatens suicide. Which criteria would enable a physician to consider involuntary commitment? (Select all that apply.) A. Being dangerous to others B. Being homeless C. Being disruptive to the community D. Being gravely disabled and unable to meet basic needs E. Being suicidal

ANS: A, D, E The physician could consider involuntary commitment when a client is being dangerous to others, is gravely disabled, or is suicidal. If the client is determined to be mentally incompetent, consent should be obtained from the legal guardian or court-approved guardian or conservator. A hospital administrator may give permission for involuntary commitment when time does not permit court intervention.

In the situation presented, which nursing intervention constitutes false imprisonment? A. The client is combative and will not redirect stating, "No one can stop me from leaving." The nurse seeks the physician's order after the client is restrained. B. The client has been consistently seeking the attention of the nurse much of the day. The nurse institutes seclusion. C. A psychotic client, admitted in an involuntary status, runs off the psychiatric unit. The nurse runs after the client and the client agrees to return. D. A client hospitalized as an involuntary admission attempts to leave the unit. The nurse calls the security team and they prevent the client from leaving.

ANS: B False imprisonment is the deliberate and unauthorized commitment of a person within fixed limits by the use of verbal or physical means. Seclusion should only be used in an emergency situation to prevent harm after least restrictive means have been unsuccessfully attempted.

The experience of being physically restrained can be traumatic. Which nursing intervention would best help the client deal with this experience? A. Administering a tranquilizing medication before applying the restraints B. Talking to the client at brief but regular intervals while the client is restrained C. Decreasing stimuli by leaving the client alone most of the time D. Checking on the client infrequently, in order to meet documentation requirements

ANS: B Restraints are never to be used as punishment or for the convenience of the staff. Connecting with the client by maintaining communication during the period of restraint will help the client recognize this intervention as a therapeutic treatment versus a punishment.

During a hiring interview, which response by a nursing applicant should indicate that the applicant operates from an ethical egoism framework? A. "I would want to be treated in a caring manner if I were mentally ill." B. "This job will pay the bills, and the workload is light enough for me." C. "I will be happy caring for the mentally ill. Working in Med/Surg kills my back." D. "It is my duty in life to be a psychiatric nurse. It is the right thing to do."

ANS: B The applicant's comment reflects an ethical egoism framework. This framework promotes the idea that decisions are made based on what is good for the individual and may not take the needs of others into account.

In response to a student's question regarding choosing a psychiatric specialty, a charge nurse states, "Mentally ill clients need special care. If I were in that position, I'd want a caring nurse also." From which ethical framework is the charge nurse operating? A. Kantianism B. Christian ethics C. Ethical egoism D. Utilitarianism

ANS: B The charge nurse is operating from a Christian ethics framework. The imperative demand of Christian ethics is to treat others as moral equals by permitting them to act as we do when they occupy a position similar to ours. Kantianism states that decisions should be made based on moral law and that actions are bound by a sense of moral duty. Utilitarianism holds that decisions should be made focusing on the end result being happiness. Ethical egoism promotes the idea that what is right is good for the individual.

At what point should the nurse determine that a client is at risk for developing a mental illness? A. When thoughts, feelings, and behaviors are not reflective of the DSM-IV-TR criteria B. When maladaptive responses to stress are coupled with interference in daily functioning C. When a client communicates significant distress D. When a client uses defense mechanisms as ego protection

ANS: B The nurse should determine that the client is at risk for mental illness when responses to stress are maladaptive and interfere with daily functioning. The DSM-IV-TR indicates that in order to be diagnosed with a mental illness, daily functioning must be significantly impaired. The client's ability to communicate distress would be considered a positive attribute.

A teenage boy is attracted to a female teacher. Without objective evidence, a school nurse overhears the boy state, "I know she wants me." This statement reflects which defense mechanism? A. Displacement B. Projection C. Rationalization D. Sublimation

ANS: B The nurse should determine that the client's statement reflects the defense mechanism of projection. Projection refers to the attribution of one's unacceptable feelings or impulses to another person. When the client "passes the blame" of the undesirable feelings, anxiety is reduced. Displacement refers to transferring feelings from one target to another. Rationalization refers to making excuses to justify behavior. Sublimation refers to channeling unacceptable drives or impulses into more constructive, acceptable activities.

A client diagnosed with paranoid schizophrenia becomes agitated when asked to play a game. The client responds, "Do you want to be my girlfriend?" Which nursing response is most appropriate? A. "You are upset now. It would be best if you go to your room until you feel better." B. "Remember, we have a professional relationship. Are you feeling uncomfortable?" C. "We have discussed this before. I am not allowed to date clients." D. "I think you should discuss your fantasies with your therapist."

ANS: B The nurse should promote the client's insight and perception of reality by confirming appropriate roles in the nurse-client relationship and identifying what is troubling the client in this situation.

A client with a history of heavy alcohol use is brought to an emergency department (ED) by family members who state that the client has had nothing to drink in the last 24 hours. Which client symptom should the nurse immediate report to the ED physician? A. Tactile hallucinations B. Blood pressure of 180/100 mm Hg C. Mood rating of 2/10 on numeric scale D. Dehydration

ANS: B The nurse should recognize that high blood pressure is a symptom of alcohol withdrawal and should promptly report this finding to the physician. Complications associated with alcohol withdrawal may progress to alcohol withdrawal delirium and possible seizure activity on about the second or third day following cessation of prolonged alcohol consumption.

A client diagnosed with post-traumatic stress disorder is admitted to an inpatient psychiatric unit for evaluation and medication stabilization. Which therapeutic communication technique used by the nurse is an example of a broad opening? A. "What occurred prior to the rape, and when did you go to the emergency department?" B. "What would you like to talk about?" C. "I notice you seem uncomfortable discussing this." D. "How can we help you feel safe during your stay here?"

ANS: B The nurse's statement, "What would you like to talk about?" is an example of the therapeutic communication technique of giving broad openings. Using a broad opening allows the client to take the initiative in introducing the topic and emphasizes the importance of the client's role in the interaction.

Which nursing action demonstrates the role of the teacher in a therapeutic milieu? A. The nurse implements a self-affirmation exercise during a one-to-one client interaction. B. The nurse holds a group meeting to present common side effects of psychiatric medications. C. The nurse introduces the concept of fair play while playing cards with a group of clients. D. The nurse models adaptive and effective coping mechanisms with clients on the psychiatric unit.

ANS: B The nurse, in the role of teacher, identifies learning needs and provides information required by the client or family to improve the client's health.

What is the priority nursing action during the orientation (introductory) phase of the nurse-client relationship? A. Acknowledge the client's actions and generate alternative behaviors B. Establish rapport and develop treatment goals C. Attempt to find alternative placement D. Explore how thoughts and feelings about this client may adversely impact care

ANS: B The priority nursing action during the orientation phase of the nurse-client relationship should be to establish rapport and develop treatment goals. Rapport implies feelings on the part of both the nurse and the client based on respect, acceptance, a sense of trust, and a nonjudgmental attitude. It is the essential foundation of the nurse-client relationship.

After fasting from 10 p.m. the previous evening, a client finds out that the blood test has been canceled. The client swears at the nurse and states, "You are incompetent!" Which is the nurse's best response? A. "Do you believe that I was the cause of your blood test being canceled?" B. "I see that you are upset, but I feel uncomfortable when you swear at me." C. "Have you ever thought about ways to express anger appropriately?" D. "I'll give you some space. Let me know if you need anything."

ANS: B This is an example of the appropriate use of feedback. Feedback should be directed toward behavior that the client has the capacity to modify.

Which nursing statement is a good example of the therapeutic communication technique of offering self? A. "I think it would be great if you talked about that problem during our next group session." B. "Would you like me to accompany you to your electroconvulsive therapy treatment?" C. "I notice that you are offering help to other peers in the milieu." D. "After discharge, would you like to meet me for lunch to review your outpatient progress?"

ANS: B This is an example of the therapeutic communication technique of offering self. Offering self makes the nurse available on an unconditional basis, increasing client's feelings of self-worth. Professional boundaries must be maintained when using the technique of offering self.

Which therapeutic communication technique should the nurse use when communicating with a client who is experiencing auditory hallucinations? A. "My sister has the same diagnosis as you and she also hears voices." B. "I understand that the voices seem real to you, but I do not hear any voices." C. "Why not turn up the radio so that the voices are muted." D. "I wouldn't worry about these voices. The medication will make them disappear."

ANS: B This is an example of the therapeutic communication technique of presenting reality. Presenting reality is when the client has a misperception of the environment. The nurse defines reality or indicates his or her perception of the situation for the client.

A client slammed a door on the unit several times. The nurse responds, "You seem angry." The client states, "I'm not angry." What therapeutic communication technique has the nurse employed and what defense mechanism is the client unconsciously demonstrating? A. Making observations and the defense mechanism of suppression B. Verbalizing the implied and the defense mechanism of denial C. Reflection and the defense mechanism of projection D. Encouraging descriptions of perceptions and the defense mechanism of displacement

ANS: B This is an example of the therapeutic communication technique of verbalizing the implied. The nurse is putting into words what the client has only implied by words or actions. Denial is the refusal of the client to acknowledge the existence of a real situation, the feelings associated with it, or both.

Which of the following behavioral skills should a nurse implement when leading a group that is functioning in the orientation phase of group development? (Select all that apply.) A. Encouraging members to provide feedback to each other about individual progress B. Ensuring that group rules do not interfere with goal fulfillment C. Working with group members to establish rules that will govern the group D. Emphasizing the need for and importance of confidentiality within the group E. Helping the members to resolve conflicts and foster cohesiveness within the group

ANS: B, C, D During the orientation phase of group development, the nurse leader should work together with members to establish rules that will effectively govern the group. The leader should ensure that group rules do not interfere with goal fulfillment and establish the need for and importance of confidentiality within the group. Members need to establish trust and cohesion to move into the working phase of group development.

A nurse attends an interdisciplinary team meeting on an inpatient unit. Which of the following individuals are typically included as members of the interdisciplinary treatment team in psychiatry? (Select all that apply.) A. Respiratory therapist and psychiatrist B. Occupational therapist and psychologist C. Recreational therapist and art therapist D. Social worker and hospital volunteer E. Mental health technician and chaplain

ANS: B, C, E The interdisciplinary treatment team in a psychiatric inpatient setting consists of a psychologist, occupational therapist, recreational therapist, art therapist, mental health technician, and chaplain. In addition, a psychiatrist, psychiatric nurse, psychiatric social worker, music therapist, psychodramatist, and dietician participate in the interdisciplinary treatment team.

How does a democratic form of self-government in the milieu contribute to client therapy? A. By setting punishments for clients who violate the community rules B. By dealing with inappropriate behaviors as they occur C. By setting community expectations wherein all clients are treated on an equal basis D. By interacting with professional staff members to learn about therapeutic interventions

ANS: C A democratic form of self-government in the milieu contributes to client therapy by setting the expectation that all clients should be treated on an equal basis. Clients participate in the decision-making and problem-solving aspects that affect treatment setting. The norms, rules, and behavioral limits are established by the staff and clients. All individuals have input.

An experienced psychiatric registered nurse has taken a new position leading groups in a day treatment program. Without further education, which group is this nurse most qualified to lead? A. A psychodrama group B. A psychotherapy group C. A parenting group D. A family therapy group

ANS: C A psychiatric registered nurse is qualified to lead a parenting group. A parenting group can be classified as either a teaching group or therapeutic group. Psychodrama, psychotherapy, and family therapy are forms of group therapy that must be facilitated by qualified leaders who generally have advanced degrees in psychology, social work, nursing, or medicine.

A client is concerned that information given to the nurse remains confidential. Which is the nurse's best response? A. "Your information is confidential. It will be kept just between you and I." B. "I will share the information with staff members only with your approval." C. "If the information impacts your care, I will need to share it with the treatment team." D. "You can make the decision whether your physician needs this information or not."

ANS: C Basic to the psychiatric client's hospitalization is his or her right to confidentiality and privacy. When admitted to an inpatient psychiatric facility, a client gives implied consent for information to be shared with health-care workers specifically involved in the client's care.

A husband accuses his wife of infidelity. Which situation would indicate to the nurse the husband's use of the ego defense mechanism of projection? A. The husband cries and stamps his feet, demanding that his wife be true to her marriage vows. B. The husband ignores the wife's continued absence from the home. C. The husband has already admitted to having an affair with a coworker. D. The husband takes out his marital frustrations through employee abuse.

ANS: C Projection is the attribution of feelings or impulses unacceptable to one's self to another person. In this situation, the husband attributes his infidelity to his wife.

Which client statement indicates that termination of the therapeutic nurse-client relationship has been handled successfully? A. "I know I can count on you for continued support." B. "I am looking forward to discharge, but I am surprised that we will no longer work together." C. "Reviewing the changes that have happened during our time together has helped me put things in perspective." D. "I don't know how comfortable I will feel when talking to someone else."

ANS: C Termination should begin in the orientation phase to minimize feelings of loss when the nurse-client relationship ends. Bringing a therapeutic conclusion to the relationship occurs when progress has been made toward attainment of mutually set goals.

A nurse moving out of state speaks to a client about the need to work with a new nurse. The client states, "You're the only one who can make me well." What does this client response indicate to the nurse? A. The client is using manipulation to receive secondary gain. B. The client is using the defense mechanism of denial. C. The client is having trouble terminating the relationship. D. The client is using "splitting" as a way to remain dependent on the nurse.

ANS: C Termination should begin in the orientation phase to minimize feelings of loss when the nurse-client relationship ends. When a client feels sadness and loss, behaviors to delay termination may become evident.

The nurse believes that a client being admitted for a surgical procedure may have a drinking problem. How should the nurse further evaluate this possibility? A. By asking directly if the client has ever had a problem with alcohol B. By holistically assessing the client using the CIWA scale C. By using a screening tool such as the CAGE questionnaire D. By referring the client for physician evaluation

ANS: C The CAGE questionnaire is a screening tool used to determine the diagnosis of alcoholism. This questionnaire is composed of four simple questions. Scoring two or three "yes" answers strongly suggests a problem with alcohol.

A client's wife has been making excuses for her alcoholic husband's work absences. In family therapy, she states, "His problems at work are my fault." Which is the appropriate nursing response? A. "Why do you assume responsibility for his behaviors?" B. "Codependency is a typical behavior of spouses of alcoholics." C. "Your husband needs to deal with the consequences of his drinking." D. "Do you understand what the term 'enabler' means?"

ANS: C The appropriate nursing response is to use confrontation with caring. The nurse should understand that the client's wife may be in denial and enabling the husband's behavior. Partners of clients with substance abuse must come to realize that the only behavior they can control is their own.

There is one bed available on an inpatient psychiatric unit. For which client should a nurse advocate emergency commitment? A. An individual who is persistently mentally ill and evicted from an apartment B. An individual treated in the emergency department (ED) for generalized anxiety disorder C. An individual who is delusional and has a plan to kill his wife D. An individual who rates mood 4/10 and is participating in a no-harm safety plan

ANS: C The criteria for involuntary emergency commitment include danger to self and/or others. Of the four clients considered, the client who is delusional and has a plan to kill his wife meets this criterion as a danger to others.

A client diagnosed with alcohol abuse joins a community 12-step program and states, "My life is unmanageable." How should the nurse interpret this client's statement? A. The client is using minimization as an ego defense. B. The client is ready to sign an Alcoholics Anonymous contract for sobriety. C. The client has accomplished the first of 12 steps advocated by Alcoholics Anonymous. D. The client has met the requirements to be designated as an Alcoholics Anonymous sponsor.

ANS: C The first step of the 12-step program advocated by Alcoholics Anonymous is that clients must admit powerlessness over alcohol and that their lives have become unmanageable.

An instructor is correcting a nursing student's clinical worksheet. Which instructor statement is the best example of effective feedback? A. "Why did you use the client's name on your clinical worksheet?" B. "You were very careless to refer to your client by name on your clinical worksheet." C. "Surely you didn't do this deliberately, but you breeched confidentiality by using the client's name." D. "It is disappointing that after being told, you're still using client names on your worksheet."

ANS: C The instructor's statement, "Surely you didn't do this deliberately, but you breeched confidentiality by using the client's name." is an example of effective feedback. Feedback is a method of communication to help others consider a modification of behavior. Feedback should be descriptive, specific, and directed toward a behavior that the person has the capacity to modify and should impart information rather than offer advice or criticize the individual.

Looking at a client's history and physical examination, a nurse notices that a diagnosis of rheumatoid arthritis has not been placed on any axis. Based on knowledge of the DSM-IV-TR, where should the nurse anticipate the coding of this assessment? A. Axis I B. Axis II C. Axis III D. Axis IV E. Axis V

ANS: C The nurse should anticipate that a diagnosis of rheumatoid arthritis should be coded on Axis III of the DSM-IV-TR diagnosis. The DSM-IV-TR uses a multiaxial system in which each axis refers to a different domain of information. Axis I should include clinical disorders and other conditions that may be a focus of clinical attention. Axis II should include information about personality disorders and mental retardation. Axis III should include information about general medication conditions. Axis IV should include information on the client's psychosocial and environmental problems. Axis V provides a Global Assessment of Functioning (GAF).

A nurse believes that the members of a parenting group are in the initial, or orientation, phase of group development. Which group behaviors would support this assumption? A. The group members manage conflict within the group. B. The group members use denial as part of the grief response. C. The group members compliment the leader and compete for the role of recorder. D. The group members initially trust one another and the leader.

ANS: C The nurse should anticipate that members in the initial, or orientation, phase of group development often compliment the leader and compete for the role of recorder. Members in this phase have not yet established trust and have a fear of not being accepted. Power struggles may occur as members compete for their position in the group.

A client has undergone psychological testing. With which member of the interdisciplinary team should a nurse collaborate to review these results? A. The psychiatrist B. The psychiatric social worker C. The clinical psychologist D. The clinical nurse specialist

ANS: C The nurse should consult with the clinical psychologist to review psychological testing results for the client. Clinical psychologists can administer, interpret, and evaluate psychological tests to assist in the diagnostic process.

Which client statement should a nurse identify as a typical response to stress most often experienced in the working phase of the nurse-client relationship? A. "I can't bear the thought of leaving here and failing." B. "I might have a hard time working with you. You remind me of my mother." C. "I really don't want to talk any more about my childhood abuse." D. "I'm not sure that I can count on you to protect my confidentiality."

ANS: C The nurse should identify that the client statement, "I really don't want to talk any more about my childhood abuse." reflects that the client is in the working phase of the nurse-client relationship. The working phase includes overcoming resistant behaviors on the part of the client as the level of anxiety rises in response to discussion of painful issues.

When planning group therapy, a nurse should identify which configuration as most optimal for a therapeutic group? A. Open-ended membership; circle of chairs; group size of 5 to 10 members B. Open-ended membership; chairs around a table; group size of 10 to 15 members C. Closed membership; circle of chairs; group size of 5 to 10 members D. Closed membership; chairs around a table; group size of 10 to 15 members

ANS: C The nurse should identify that the most optimal conditions for a therapeutic group are when the membership is closed and the group size is between 5 and 10 members who are arranged in a circle of chairs. The focus of therapeutic groups is on relationships within the group and the interactions among group members.

During group therapy, a client diagnosed with chronic alcohol dependence states, "I would not have boozed it up if my wife hadn't been nagging me all the time to get a job. She never did think that I was good enough for her." How should a nurse interpret this statement? A. The client is using denial by avoiding responsibility. B. The client is using displacement by blaming his wife. C. The client is using rationalization to excuse his alcohol dependence. D. The client is using reaction formation by appealing to the group for sympathy.

ANS: C The nurse should interpret that the client is using rationalization to excuse his alcohol dependence. Rationalization is the defense mechanism by which people avoid taking responsibility for their actions by making excuses for the behavior.

A client who will be receiving electroconvulsive therapy (ECT) must provide informed consent. Which situation should cause a nurse to question the validity of the informed consent? A. The client is paranoid. B. The client is 87 years old. C. The client incorrectly reports his or her spouse's name, date, and time of day. D. The client relies on his or her spouse to interpret the information.

ANS: C The nurse should question the validity of informed consent when the client incorrectly reports the spouse's name, date, and time of day. This indicates that this client is disoriented and may not be competent to make informed choices.

Devastated by a divorce from an abusive husband, a wife completes grief counseling. Which statement by the wife should indicate to a nurse that the client is in the acceptance stage of grief? A. "If only we could have tried again, things might have worked out." B. "I am so mad that the children and I had to put up with him as long as we did." C. "Yes, it was a difficult relationship, but I think I have learned from the experience." D. "I still don't have any appetite and continue to lose weight."

ANS: C The nurse should recognize that the client is in the acceptance stage of grief. During this stage of the grief process, the client would be able to focus on the reality of the loss and its meaning in relation to life.

A nurse is caring for a client who is status post-coronary artery bypass surgery and has recently been diagnosed with major depressive disorder. A nurse should recognize that the DSM-IV-TR places these diagnoses on which two axes? A. Axes I and IV B. Axes II and V C. Axes I and III D. Axes III and V

ANS: C The nurse should understand that clinical disorders, such as major depressive disorder, should be included in Axis I of the DSM-IV-TR. The DSM-IV-TR uses a multiaxial system in which each axis refers to a different domain of information. Axis I should include clinical disorders and other conditions that may be a focus of clinical attention. Axis II should include information about personality disorders and mental retardation. Axis III should include information about general medication conditions like a recent history of coronary artery bypass surgery. Axis IV should include information on the client's psychosocial and environmental problems. Axis V provides a Global Assessment of Functioning (GAF).

A client diagnosed with schizophrenia refuses to take medication, citing the right of autonomy. Under which circumstance would a nurse have the right to medicate the client against the client's wishes? A. When the client makes inappropriate sexual innuendos to a staff member B. When the client constantly demands inappropriate attention from the nurse C. When the client physically attacks another client after being confronted in group therapy D. When the client refuses to bathe or perform hygienic activities

ANS: C The nurse would have the right to medicate a client against his or her wishes if the client physically attacks another client. This client poses a significant risk to safety and is incapable of making rational choices. The client's refusal to accept treatment can be challenged because the client is endangering the safety of others.

An angry client on an inpatient unit approaches a nurse stating, "Someone took my lunch! People need to respect others, and you need to do something about this now!" The nurse's response should be guided by which basic assumption of milieu therapy? A. Conflict should be avoided at all costs on inpatient psychiatric units. B. Conflict should be resolved by the nursing staff. C. Every interaction is an opportunity for therapeutic intervention. D. Conflict resolution should only be addressed during group therapy.

ANS: C The nurse's response should be guided by the basic assumption that every interaction is an opportunity for therapeutic intervention. The nurse can utilize milieu therapy to effect behavioral change and improve psychological health and functioning.

The nurse asks a newly admitted client, "What can we do to help you?" What is the purpose of this therapeutic communication technique? A. To reframe the client's thoughts about mental health treatment B. To put the client at ease C. To explore a subject, idea, experience, or relationship D. To communicate that the nurse is listening to the conversation

ANS: C This is an example of the therapeutic communication technique of exploring. The purpose of using exploring is to delve further into the subject, idea, experience, or relationship. This technique is especially helpful with clients who tend to remain on a superficial level of communication.

Which nursing statement is a good example of the therapeutic communication technique of giving recognition? A. "You did not attend group today. Can we talk about that?" B. "I'll sit with you until it is time for your family session." C. "I notice you are wearing a new dress and you have washed your hair." D. "I'm happy that you are now taking your medications. They will really help."

ANS: C This is an example of the therapeutic communication technique of giving recognition. Giving recognition acknowledges and indicates awareness. This technique is more appropriate than complimenting the client which reflects the nurse's judgment.

A client on an inpatient psychiatric unit tells the nurse, "I should have died because I am totally worthless." In order to encourage the client to continue talking about feelings, which should be the nurse's initial response? A. "How would your family feel if you died?" B. "You feel worthless now, but that can change with time." C. "You've been feeling sad and alone for some time now?" D. "It is great that you have come in for help."

ANS: C This nursing statement is an example of the therapeutic communication technique of reflection. When reflection is used, questions and feelings are referred back to the client so that they may be recognized and accepted.

A client with a history of insomnia has been taking chlordiazepoxide (Librium) 15 mg at night for the past year. The client currently reports getting to sleep. Which nursing diagnosis appropriately documents this problem? A. Ineffective coping R/T unresolved anxiety AEB substance abuse B. Anxiety R/T poor sleep AEB difficulty falling asleep C. Disturbed sleep pattern R/T Librium tolerance AEB difficulty falling asleep D. Risk for injury R/T addiction to Librium

ANS: C Tolerance is defined as the need for increasingly larger or more frequent doses of a substance in order to obtain the desired effects originally produced by a lower dose.

When interviewing a client, which nonverbal behavior should a nurse employ? A. Maintaining indirect eye contact with the client B. Providing space by leaning back away from the client C. Sitting squarely, facing the client D. Maintaining open posture with arms and legs crossed

ANS: C When interviewing a client, the nurse should employ the nonverbal behavior of sitting squarely, facing the client. Facilitative skills for active listening can be identified by the acronym SOLER. SOLER includes sitting squarely facing the client (S), open posture when interacting with a client (O), leaning forward toward the client (L), establishing eye contact (E), and relaxing (R).

As the client and nurse move from the orientation stage to the working stage of the therapeutic relationship, which is the nurse's most therapeutic statement? A. "I want to assure you that I will maintain your confidentiality." B. "A long-term goal for someone your age would be to develop better job skills." C. "Which identified problems would you like for us to initially address?" D. "I think first we need to focus on your relationship issues."

ANS: C When moving on a continuum from the orientation to working phase of the nurse-client relationship, identified goals are addressed through mutual therapeutic work to promote client behavioral change.

Prayer group members at a local Baptist church are meeting with a poor, homeless family they are supporting. Which member statement is an example of Yalom's curative group factor of altruism? A. "I'll give you the name of a friend that rents inexpensive rooms." B. "The last time we helped a family, they got back on their feet and prospered." C. "I can give you all of my baby clothes for your little one." D. "I can appreciate your situation. I had to declare bankruptcy last year."

ANS: C Yalom's curative group factor of altruism occurs when group members provide assistance and support to each other creating a positive self-image and promoting self-growth. Individuals increase self-esteem through mutual caring and concern.

According to Peplau, which nursing action demonstrates the nurse's role as a resource person? A. The nurse balances a safe therapeutic environment to increase the client's sense of belonging. B. The nurse holds a group meeting with the clients on the unit to discuss common feelings about mental illness. C. The nurse monitors the administration of medications and watches for signs of "cheeking." D. The nurse explains, in language the client can understand, information related to the client's health care.

ANS: D According to Peplau, a resource person provides specific answers to questions usually formulated with relation to a larger problem.

Which is an accurate description of a common law? A. A common law would be invoked to deal with a nurse who, without justification, threatens a client with restraints. B. A common law would be invoked to deal with a nurse who touches a client without the client's consent. C. A common law would be invoked to deal with a hospital employee who steals drugs, hospital equipment, or both. D. A common law would be invoked to deal with a nurse's refusal to provide care for a specific client.

ANS: D Common laws apply to a body of principles that evolve from court decisions resolving various controversies. Common law may vary from state to state. Assault (threats) and battery (touch) are governed by civil law. Stealing is governed by criminal law.

A nurse is caring for a client who has been prescribed disulfiram (Antabuse) as a deterrent to alcohol relapse. Which information should the nurse include when teaching the client about this medication? A. "Only oral ingestion of alcohol will cause a reaction when taking this drug." B. "It is safe to drink beverages that have only 12% alcohol content." C. "This medication will decrease your cravings for alcohol." D. "Reactions to combining Antabuse with alcohol can occur 2 weeks after stopping the drug."

ANS: D If Antabuse is discontinued, it is important for the client to understand that the sensitivity to alcohol may last for as long as 2 weeks.

An involuntarily committed client is verbally abusive to the staff and repeatedly threatening to sue. The client records the full names and phone numbers of the staff. Which nursing action is most appropriate to decrease the possibility of a lawsuit? A. Verbally redirect the client, and then limit one-on-one interaction. B. Involve the hospital's security division as soon as possible. C. Notify the client that documenting personal staff information is against hospital policy. D. Continue professional attempts to establish a positive working relationship with the client.

ANS: D The most appropriate nursing action is to continue professional attempts to establish a positive working relationship with the client. The involuntarily committed client should be respected and has the right to assert grievances if rights are infringed.

A client on an inpatient unit angrily states to a nurse, "Peter is not cleaning up after himself in the community bathroom. You need to address this problem." Which is the appropriate nursing response? A. "I'll talk to Peter and present your concerns." B. "Why are you overreacting to this issue?" C. "You should bring this to the attention of your treatment team." D. "I can see that you are angry. Let's discuss ways to approach Peter with your concerns."

ANS: D The most appropriate nursing response involves restating the client's feeling and developing a plan with the client to solve the problem. According to Skinner, every interaction in the therapeutic milieu is an opportunity for therapeutic intervention to improve communication and relationship-development skills.

A newly admitted client diagnosed with obsessive-compulsive disorder (OCD) washes hands continually. This behavior prevents unit activity attendance. Which nursing statement best addresses this situation? A. "Everyone diagnosed with OCD needs to control their ritualistic behaviors." B. "It is important for you to discontinue these ritualistic behaviors." C. "Why are you asking for help if you won't participate in unit therapy?" D. "Let's figure out a way for you to attend unit activities and still wash your hands."

ANS: D The most appropriate statement by the nurse is, "Let's figure out a way for you to attend unit activities and still wash your hands." This statement reflects the therapeutic communication technique of formulating a plan of action. The nurse attempts to work with the client to develop a plan without damaging the therapeutic relationship or increasing the client's anxiety.

Which group leader activity should a nurse identify as being most effective in the final, or termination, phase of group development? A. The group leader establishes the rules that will govern the group after discharge. B. The group leader encourages members to rely on each other for problem solving. C. The group leader presents and discusses the concept of group termination. D. The group leader helps the members to process feelings of loss.

ANS: D The most effective intervention in the final, or termination, phase of group development would be for the group leader to help the members to process feelings of loss. The leader should encourage the members to review the goals and discuss outcomes, reminisce about what has occurred, and encourage members to provide feedback to each other about progress.

A client who frequently exhibits angry outbursts is diagnosed with antisocial personality disorder. Which appropriate feedback should a nurse provide when this client experiences an angry outburst? A. "Why do you continue to alienate your peers by your angry outbursts?" B. "You accomplish nothing when you lose your temper like that." C. "Showing your anger in that manner is very childish and insensitive." D. "During group, you raised your voice, yelled at a peer, left, and slammed the door."

ANS: D The nurse is providing appropriate feedback when stating, "During group, you raised your voice, yelled at a peer, left, and slammed the door." Giving appropriate feedback involves helping the client consider a modification of behavior. Feedback should give information to the client about how he or she is perceived by others. Feedback should not be evaluative in nature or be used to give advice.

A nurse is assessing a client who is experiencing occasional feelings of sadness because of the recent death of a beloved pet. The client's appetite, sleep patterns, and daily routine have not changed. How should the nurse interpret the client's behaviors? A. The client's behaviors demonstrate mental illness in the form of depression. B. The client's behaviors are extensive which indicates the presence of mental illness. C. The client's behaviors are not congruent with cultural norms. D. The client's behaviors demonstrate no functional impairment, indicating no mental illness.

ANS: D The nurse should assess that the client's daily functioning is not impaired. The client who experiences feelings of sadness after the loss of a pet is responding within normal expectations. Without significant impairment, the client's distress does not indicate a mental illness.

An inpatient psychiatric physician refuses to treat clients without insurance and prematurely discharges those whose insurance benefits have expired. Which violation of an ethical principle should a nurse recognize in this situation? A. Autonomy B. Beneficence C. Nonmaleficence D. Justice

ANS: D The nurse should determine that the ethical principle of justice has been violated by the physician's actions. The principle of justice requires that individuals should be treated equally regardless of race, sex, marital status, medical diagnosis, social standing, economic level, or religious belief.

What is the purpose of a nurse providing appropriate feedback? A. To give the client good advice B. To advise the client on appropriate behaviors C. To evaluate the client's behavior D. To give the client critical information

ANS: D The purpose of providing appropriate feedback is to give the client critical information. Feedback should not be used to give advice or evaluate behaviors.

During the sixth week of a 10-week parenting skills group, a nurse observes as several members get into a heated dispute about spanking. As a group, they decide to create a pros-and-cons poster on the use of physical discipline. At this time, what is the role of the group leader? A. To referee the debate B. To adamantly oppose physical discipline measures C. To redirect the group to a less controversial topic D. To encourage the group to solve the problem collectively

ANS: D The role of the group leader is to encourage the group to solve the problem collectively. A democratic leadership style supports members in their participation and problem solving. Members are encouraged to cooperatively solve issues that relate to the group.

A single, pregnant teenager in a parenting class discloses her ambivalence toward the pregnancy and the subsequent guilt that these thoughts generate. A mother of three admits to having felt that way herself. Which of Yalom's curative group factors does this illustrate? A. Imparting of information B. Instillation of hope C. Altruism D. Universality

ANS: D The scenario is an example of the curative group factor of universality. Universality occurs when individuals realize that they are not alone in the problems, thoughts, and feelings they are experiencing. This realization reduces anxiety by the support and understanding of others.

A nursing instructor is teaching students about self-help groups like Alcoholics Anonymous (AA). Which student statement indicates that learning has occurred? A. "There is little research to support AA's effectiveness." B. "Self-help groups used to be the treatment of choice, but their popularity is waning." C. "These groups have no external regulation, so clients need to be cautious." D. "Members themselves run the group, with leadership usually rotating among the members."

ANS: D The student indicates an understanding of self-help groups when stating, "Members themselves run the group, with leadership usually rotating among the members." Nurses may or may not be involved in self-help groups. These groups allow members to talk about feelings and reduce feelings of isolation while receiving support from others undergoing similar experiences.

A nurse is interviewing a client in an outpatient substance-abuse clinic. To promote success in the recovery process, which outcome should the nurse expect the client to initially accomplish? A. The client will identify one person to turn to for support. B. The client will give up all old drinking buddies. C. The client will be able to verbalize the effects of alcohol on the body. D. The client will correlate life problems with alcohol use.

ANS: D To promote the recovery process the nurse should expect that the client would initially correlate life problems with alcohol use. Acceptance of the problem is the first step of the recovery process.

b Strong positive or negative reactions to a patient or overidentification with the patient signals possible countertransference. Nurses must carefully monitor their own feelings and reactions to detect countertransference, then seek supervision. The distracters identify desirable outcomes or transference.

After several therapeutic encounters with a patient who recently attempted suicide, which behavior should cause the nurse to consider the possibility of countertransference? a. The patient's reactions toward the nurse seem realistic and appropriate. b. The nurse feels very happy when the patient's mood begins to lift. c. The patient states, "Talking to you is like talking to my parents." d. The nurse develops a trusting relationship with the patient.

d Sullivan's theory explains that security operations are interpersonal relationship activities designed to relieve anxiety. Because they are interpersonal, they are observable. Defense mechanisms are unconscious and automatic. Repression is entirely intrapsychic, but other mechanisms result in observable behaviors. Frequent, continued use of many defense mechanisms often results in reality distortion and interference with healthy adjustment and emotional development. Occasional use of defense mechanisms is normal and does not markedly interfere with development. Security operations are ego centered.

Although ego defense mechanisms and security operations are unconsciously determined and designed to relieve anxiety, the major difference is: a. defense mechanisms are intrapsychic and not observable. b. defense mechanisms cause arrested personal development. c. security operations are masterminded by the id and superego. d. security operations address interpersonal relationship activities.

b

As L converses with the nurse, she states, "I dreamed I was stoned. When I woke up, I was feeling emotionally drained as though I hadn't rested well." If the nurse needs clarification of "stoned," it would be appropriate to say, a. "It sounds as though you were quite uncomfortable with the content of your dream." b. "Can you give me an example of what you mean by stoned?" c. "I understand what you're saying. Bad dreams leave me feeling tired, too." d. "So, all in all, you feel as though you had a rather poor night's sleep?"

b The invitation creates a social rather than therapeutic relationship. It may or may not increase dependency on the nurse.

As a patient with mental illness is discharged from a facility, the nurse invites the patient to the annual staff holiday party. Select the best analysis of this scenario. a. The invitation facilitates dependency on the nurse. b. The nurse's action blurs the boundaries of the therapeutic relationship. c. The invitation is therapeutic for the patient's diversional activity deficit. d. The nurse's action assists the patient's integration into community living.

a The theory of interpersonal relationships recognizes the anxiety and depression as resulting from unmet interpersonal security needs. Behaviorism and classical conditioning theories do not apply. A psychosexual formulation would focus on uncovering unconscious material that relates to the patient problem.

Consider a therapist's statement: "The patient is homosexual but has kept this preference secret. Severe anxiety and depression occur when the patient anticipates family reactions to this sexual orientation." Which perspective is evident in the speaker? a. Theory of interpersonal relationships b. Classical conditioning theory c. Psychosexual theory d. Behaviorism theory

a

For the client whose nursing diagnosis is powerlessness related to inability to control compulsive cleaning, the nurse recognizes that the client uses the cleaning to a. temporarily reduce anxiety b. gain a feeling of superiority c. receive praise from friends and family d. ensure the health of household members

b A patient has the right to know with whom the nurse will share information and that confidentiality will be protected. Although the relationship is primarily between the nurse and patient, other staff needs to know pertinent data. The relationship must be patient centered and have clear boundaries.

How should the nurse respond if a patient says, "Please don't share information about me with the other people"? a. "I cannot tell anyone about you. We can help each other by keeping it between us." b. "I won't share information with your family or friends without your permission, but I will share information with other staff." c. "It depends on what you choose to tell me. I will be glad to disclose at the end of each session what I will report to other staff." d. "Therapeutic relationships are between the nurse and the patient. It's up to you to tell others what you want them to know."

d The nurse is concerned about justice, that is, fair distribution of care, which includes treatment with the least restrictive methods for both patients. Beneficence means promoting the good of others. Autonomy is the right to make one's own decisions. Fidelity is the observance of loyalty and commitment to the patient.

In a team meeting a nurse says, "I'm concerned about whether we are behaving ethically in using restraint to prevent one patient from engaging in self-mutilating behavior while the care plan for another self-mutilating patient calls for one-on-one supervision." Which ethical principle most clearly applies to this situation? a. Beneficence b. Autonomy c. Fidelity d. Justice

a The question asks about risk. Hearing voices is generally associated with mental illness, but in charismatic religious groups, hearing the voice of God or a prophet is a desirable event. Cultural norms vary, which makes it more difficult to make an accurate diagnosis. The individuals described in the other options are less likely to be labeled mentally ill.

In the majority culture of the United States, which individual has the greatest risk to be labeled mentally ill? One who: a. describes hearing God's voice speaking. b. is usually pessimistic but strives to meet personal goals. c. is wealthy and gives away $20 bills to needy individuals. d. always has an optimistic viewpoint about life and having own needs met.

d The medical diagnosis is concerned with the patient's disease state, causes, and cures, whereas the nursing diagnosis focuses on the patient's response to stress and possible caring interventions. Both tools consider culture. The DSM-IV-TR is multiaxial. Nursing diagnoses also consider potential problems.

Select the best response for the nurse who receives a query from another mental health professional seeking to understand the difference between a DSM-IV-TR diagnosis and a nursing diagnosis. a. "There is no functional difference between the two. Both identify human disorders." b. "The DSM-IV-TR diagnosis disregards culture, whereas the nursing diagnosis takes culture into account." c. "The DSM-IV-TR diagnosis is associated with present distress or disability, whereas a nursing diagnosis considers past and present responses to actual mental health problems." d. "The DSM-IV-TR diagnosis affects the choice of medical treatment, whereas the nursing diagnosis offers a framework for identifying interventions for phenomena a patient is experiencing."

b Development of rapport and trust is necessary before the relationship can progress to the working phase. The distracters present outcomes associated with the working phase.

Select the desirable outcome for the initial stage of a nurse-patient relationship. The patient will demonstrate behaviors that indicate: a. a greater sense of independence. b. rapport and trust with the nurse. c. resolved transference. d. self-responsibility.

b A tort is a civil wrong against a person that violates his or her rights. Giving unnecessary medication for the convenience of staff controls behavior in a manner similar to secluding a patient; thus false imprisonment is a possible charge. The other options do not exemplify torts.

Select the example of a tort. a. The plan of care for a patient is not completed within 24 hours of the patient's admission. b. A nurse gives a PRN dose of an antipsychotic drug to an agitated patient because the unit is short staffed. c. An advanced practice nurse recommends hospitalization for a patient who is dangerous to self and others. d. A patient's admission status changed from involuntary to voluntary after the patient's hallucinations subside.

d The DSM-IV-TR classifies disorders people have rather than people themselves. The terminology of the tool reflects this distinction by referring to individuals with a disorder rather than as a "schizophrenic" or "alcoholic," for example. Deviant behavior is not generally considered a mental disorder. Present disability or distress is only one aspect of the diagnosis.

The Diagnostic and Statistical Manual of Mental Disorders classifies: a. deviant behaviors. b. people with mental disorders. c. present disability or distress. d. mental disorders people have.

a The right to medical and psychiatric treatment was conferred on all patients hospitalized in public mental hospitals with the enactment of the federal Hospitalization of Mentally Ill Act in 1964.

The family of a patient whose insurance will not pay for continuing hospitalization considers transferring the patient to a public mental hospital. They express concern that the patient will be "never get any treatment." Which reply by the nurse would be most helpful? a. "Under the law, treatment must be provided. Hospitalization without treatment violates patients' rights." b. "All patients in public hospitals have the right to choose both a primary therapist and a primary nurse." c. "That's a justifiable concern because the right to treatment extends only to provision of food, shelter, and safety." d. "Much will depend on other patients, because the right to treatment for a psychotic patient takes precedence over the right to treatment of a patient who is stable."

b A nurse's negative preconceived ideas about a patient and negative feelings toward the patient usually result in frustration and mutual withdrawal. Supervision would cause the nurse to explore the origins of the feelings and make changes as necessary. The other options suggest positive outcomes, which rarely occur in the face of negative feelings on the part of the nurse.

The nurse tells a peer, "I feel very uncomfortable with a patient and find myself wanting to avoid both informal contacts and scheduled sessions." Without supervision, which outcome is likely? a. Growing interest and mutuality b. Mutual withdrawal c. Positive regard d. Trust

b Mood changes throughout the day may be related to circadian rhythm disturbances. Questions about sleep pattern are also relevant to circadian rhythms. The distracters apply to assessment for illusions and hallucinations, thought processes, and memory.

The nurse wants to assess a patient with major depression for disturbances in circadian rhythms. Select the best question for this aspect of the assessment. a. "Have you ever seen or heard things that others do not?" b. "What are your worst and best times of day?" c. "How would you describe your thinking?" d. "Do you think your memory is failing?"

c The superego contains the "thou shalts," or moral standards internalized from interactions with significant others. Praise fosters internalization of desirable behaviors. The id is the center of basic instinctual drives, and the ego is the mediator. The ego is the problem-solving and reality-testing portion of the personality that negotiates solutions with the outside world. The preconscious is a level of awareness from which material can be retrieved easily with conscious effort.

The parent of a 4-year-old rewards and praises the child for helping a younger sibling, being polite, and using good manners. The nurse supports the use of praise related to these behaviors. These qualities will likely be internalized and become part of the child's: a. id. b. ego. c. superego. d. preconscious.

b The parent's comment suggests feelings of guilt or inadequacy. The nurse's response should address these feelings as well as provide information. Patients and families need reassurance that the major mental disorders are biological in origin and are not the "fault" of parents. One distracter places the burden of having faulty genes on the shoulders of the parents. The other distracters are neither wholly accurate nor reassuring.

The parent of a child with schizophrenia tearfully asks the nurse, "What could I have done differently to prevent this illness?" Select the nurse's best response. a. "Although schizophrenia results from impaired family relationships, try not to feel guilty. No one can predict how a child will respond to parental guidance." b. "Schizophrenia is a biological illness resulting from changes in how the brain and nervous system function. You are not to blame for your child's illness." c. "There is still hope. Changing your parenting style can help your child learn to cope effectively with the environment." d. "Most mental illnesses result from genetic inheritance. Your genes are more at fault than your parenting."

b Axis III indicates any relevant general medical conditions. Axis II refers to personality disorders and mental retardation. Together they constitute the classification of abnormal behavior diagnosed in the individual. Axis IV reports psychosocial and environmental problems that may affect the diagnosis, treatment, and prognosis. Axis V is the global assessment of functioning.

The psychiatric nurse addresses axis I of the DSM as the focus of treatment but must also consider physical health problems that may affect treatment. Which axis contains the desired information? a. II b. III c. IV d. V

b If the reuptake of a substance is inhibited, it accumulates in the synaptic gap and its concentration increases, permitting ease of transmission of impulses across the synaptic gap. Normal transmission of impulses across synaptic gaps is consistent with normal rather than depressed mood. The other options are not associated with blocking neurotransmitter reuptake.

The therapeutic action of neurotransmitter inhibitors that block reuptake cause: a. decreased concentration of the neurotransmitter in the central nervous system. b. increased concentration of neurotransmitter in the synaptic gap. c. destruction of receptor sites. d. limbic system stimulation.

b Patients have a right to treatment in the least restrictive setting. Safety is important, but less restrictive measures should be tried first. Unnecessary seclusion may result in a charge of false imprisonment. Seclusion removes the patient's autonomy. The principle by which the nurse is motivated is beneficence, not justice. The tort represented is false imprisonment

Two hospitalized patients fight when they are in the same room. During a team meeting, a nurse asserts that safety is of paramount importance, so treatment plans should call for both patients to be secluded to keep them from injuring each other. This assertion: a. reinforces the autonomy of the two patients. b. violates the civil rights of both patients. c. represents the intentional tort of battery. d. correctly places emphasis on safety.

c

Two staff nurses were considered for promotion. The promotion was announced via a memo on the unit bulletin board. The nurse who was not promoted told another friend, "I knew I'd never get the job. The hospital administrator hates me." If she actually believes this of the administrator, who, in reality, knows little of her, she is demonstrating a. Compensation b. Reaction formation c. Projection d. Denial

d

W has a mass in the left upper lobe of his lung. He is scheduled to undergo a biopsy. When the nurse explains the procedure to him, he seems to have difficulty grasping what she is saying and asks questions such as, "What do you mean I'm going to have surgery? What are they going to do?" His voice is tremulous. His respirations are noticeably rapid at 28 breaths per minute, and his pulse is 110 beats per minute. W can be assessed as having a cognitive problem called a. Rationalization b. Conversion c. Introjection d. selective inattention

c

What behavior on the part of Nurse G will produce the evaluation that termination of the therapeutic nurse-client relationship with P, a client, has been handled successfully? a. He gives P his personal telephone number and permission to call after her discharge. b. He avoids upsetting P by gradually focusing on other clients beginning 1 week prior to her discharge. c. He summarizes with P the changes that have happened during their time together and evaluates goal attainment. d. He offers to meet P for coffee and conversation three times a week for 2 weeks after her discharge.

a A simple way of showing respect is to address the patient by title and surname rather than assume the patient would wish to be called by the first name. The distracters violate confidentiality and autonomy or exemplify beneficence and fidelity.

Which action by a psychiatric nurse best supports the right of patients to be treated with dignity and respect? a. Consistently addresses patients by title and surname b. Strongly encourages a patient to participate in the unit milieu c. Discusses a patient's condition with the health care provider in the elevator d. Informs a treatment team that a patient is too drowsy to participate in care planning

d A nurse who understands that a patient's symptoms are influenced by culture will be able to advocate for the patient to a greater degree than a nurse who believes that culture is of little relevance. The distracters are untrue statements.

Which belief will best support a nurse's efforts to provide patient advocacy during a multidisciplinary patient care planning session? a. All mental illnesses are culturally determined. b. Schizophrenia and bipolar disorder are cross-cultural disorders. c. Symptoms of mental disorders are unchanged from culture to culture. d. Assessment findings in mental disorders reflect a person's cultural patterns.

b Stigma refers to stereotypical, negative beliefs. With respect to mental health and mental illness, stigma often leads to discrimination and uncaring attitudes. Mental illness has multiple causes, including stress, changes in brain structure or function, and genetic transmission.

Which comment most clearly shows a speaker views mental illness with stigma? a. "Some mental illnesses are inherited." b. "Most people with mental illness are unmotivated." c. "Severe environmental stress sometimes causes mental illness." d. "Some mental illnesses are brain disorders resulting from changes in how impulses are transmitted."

d The documentation states specific observations of the patient's appearance and the exact statements made. The other options are vague or subjective statements and can be interpreted in different ways.

Which documentation of a patient's behavior best demonstrates a nurse's observations? a. Isolates self from others. Frequently fell asleep during group. Vital signs stable. b. Calmer; more cooperative. Participated actively in group. No evidence of psychotic thinking. c. Appeared to hallucinate. Frequently increased volume on television, causing conflict with others. d. Wore four layers of clothing. States, "I need protection from evil bacteria trying to pierce my skin."

d The DSM-IV-TR profiles psychiatric diagnoses on five axes. Each axis defines a specific aspect of the diagnosis. Axis I identifies major clinical disorders. Axis II details personality and developmental disorders. Axis III identifies general medical conditions. Axis IV details psychosocial and environmental problems. Axis V rates the Global Assessment of Functioning.

Which documentation of diagnosis would a nurse expect in a psychiatric treatment setting? a. I Acute renal failure II 75 III Bipolar disorder I, mixed IV Loss of disability benefits 2 months ago V None b. I Schizophrenia, paranoid type II Death of spouse last year III 60 IV None V Diabetes, type 2 c. I Polysubstance dependence II Narcissistic Personality Disorder III 90 IV Hyperlipidemia V Charges pending for assault d. I Major Depression II Avoidant Personality Disorder III Hypertension IV Home destroyed by hurricane last year V 80

a The correct response describes an adaptive, healthy behavior. The distracters describe maladaptive behaviors.

Which finding best indicates that the goal "Demonstrate mentally healthy behavior" was achieved? A patient: a. sees self as approaching ideals and capable of meeting demands. b. seeks others to assume responsibility for major areas of own life. c. behaves without considering the consequences of personal actions. d. aggressively meets own needs without considering the rights of others.

d Throwing a heavy plate is likely to harm the waiter and is evidence of dangerousness to others. This behavior meets the criteria for emergency or involuntary hospitalization for mental illness. The behaviors in the other options evidence mental illness but not dangerousness.

Which individual with mental illness may need emergency or involuntary hospitalization for mental illness? The individual who: a. resumes using heroin while still taking naltrexone (ReVia). b. reports hearing angels playing harps during thunderstorms. c. does not keep an outpatient appointment with the mental health nurse. d. throws a heavy plate at a waiter at the direction of command hallucinations.

a Autonomy is the right to self-determination, that is, to make one's own decisions. By exploring alternatives with the patient, the patient is better equipped to make an informed, autonomous decision. The distracters demonstrate beneficence, fidelity, and justice.

Which intervention by a psychiatric nurse best applies the ethical principle of autonomy? The nurse: a. explores alternative solutions with a patient, who than makes a choice. b. suggests that two patients who were fighting be restricted to the unit. c. intervenes when a self-mutilating patient attempts to harm self. d. stays with a patient demonstrating a high level of anxiety.

c Relationship parameters, the contract, confidentiality, and termination should be considered during the orientation phase of the relationship. The distracters represent issues dealt with later.

Which issues should a nurse address during the first interview with a patient with a psychiatric disorder? a. Trust, congruence, attitudes, and boundaries. b. Goals, resistance, unconscious motivations, and diversion. c. Relationship parameters, the contract, confidentiality, and termination. d. Transference, countertransference, intimacy, and developing resources.

b False imprisonment involves holding a competent person against his or her will. Actual force is not a requirement of false imprisonment. The individual needs only to be placed in fear of imprisonment by someone who has the ability to carry out the threat. If a patient is not competent (confused), the nurse should act with beneficence. Patients admitted involuntarily should not be allowed to leave without permission of the treatment team.

Which nursing intervention demonstrates false imprisonment? a. A confused and combative patient says, "I'm getting out of here and no one can stop me." The nurse restrains this patient without a health care provider's order and then promptly obtains an order. b. A patient has been irritating and attention seeking much of the day. Now a nurse escorts the patient down the hall saying, "Stay in your room or you'll be put in seclusion." c. An involuntarily hospitalized patient with suicidal ideation runs out of the psychiatric unit. The nurse rushes after the patient and convinces the patient to return to the unit. d. An involuntarily hospitalized patient with homicidal ideation attempts to leave the facility. A nurse calls the security team and uses established protocols to prevent the patient from leaving.

d Involuntary commitment protects patients who are dangerous to themselves or others and cannot care for their own basic needs. Involuntary commitment also protects other individuals in society. The behaviors described in the other options are not sufficient to require involuntary hospitalization.

Which patient meets criteria for involuntary commitment for psychiatric treatment? The patient who: a. is noncompliant with the treatment regimen. b. fraudulently files for bankruptcy. c. sold and distributed illegal drugs. d. threatens to harm self and others.

b According to Erikson, the developmental task of infancy is the development of trust. The correct response is the only statement clearly showing lack of ability to trust others. Warm, close relationships suggest the developmental task of infancy was successfully completed; rigidity and self-absorption are reflected in the belief one is always right; and shame for past actions suggests failure to resolve the crisis of initiative versus guilt.

Which patient statement would lead the nurse to suspect unsuccessful completion of the developmental task of infancy? a. "I have very warm and close friendships." b. "I'm afraid to allow anyone to really get to know me." c. "I'm always absolutely right, so don't bother saying more." d. "I'm ashamed that I didn't do things correctly in the first place."

d Thinking about a more constructive approach to dealing with anger indicates a readiness to make a behavioral change. Behavioral change is associated with the working phase of the relationship. Denial or avoidance are often seen in the orientation phase.

Which remark by a patient indicates movement from orientation to the working phase of a nurse-patient relationship? a. "I don't have any problems." b. "It is so difficult for me to talk about problems." c. "I don't know how talking about things twice a week can help." d. "I want to find ways to deal with my anger without blowing up."

____ is a mental illness with symptoms that generally reflect a progressive deterioration and disorganization of the individual's personality structure, affect, and cognition.

schizophrenia

Which intervention is not considered part of the psychotherapeutic management model in the continuum of care?

self-help groups the model consists of nurse patient relationship, psychopharmacology, and milieu manaement

What was a key factor that motivated the passage of the Community Mental Health Centers Act of 1963?

that mentally ill individuals had been geographically isolated from family and community

What is the highest chain when following the chain of command in statutory law?

the Constitution of the United States

What was the first asylum in the US?

the Eastern Lunatic Asylum in Williamsburg, VA founded in 1773

Which of Freud's contributions to psychiatry most affects current psychiatric nursing?

the challenge to look at humans objectively

issues that affect the delivery of psychiatric care

the paradigm shift in psychiatric care homelessness: 25% of homeless have a severe mental illness, 54% of women and 84% of men suffer from alcohol or drug abuse, or both, the behavior from aggressive panhandling t public elimination of bodily wastes has alienated mainstream america

What rights does the person lose once they are judged incompetent?

the right to marry, vote, drive a car, and enter into contracts

What does the DSM outline?

the signs and symptoms required in order for clinicians to assign a specific diagnosis to a patient

What is false imprisonment?

the unlawful restraint of an individuals personal liberty or unlawful restraint or confinement of an individual

epidemiology of mental disorders

twenty five percent of American adults over 17 years old meet the criteria for a mental disorder during any 12 month period, 50% of the individuals with a mental disorder experience the illness over a lifetime, major mental disorders that cause disability include major depression, schizophrenia, bipolar disorder, and alcohol abuse, one half of all mental disorders begin in the teen years


Conjuntos de estudio relacionados

dance is an activity which can take many forms and different needs .it can be recreation, entertainment education, therapy and religion.pep

View Set

Osteoporosis NCLEX Style Questions

View Set

6.3 - Triumph of Parliament in England

View Set

Ch. #4 Multicultural Perspectives and Diversity Issues

View Set

Probability (Independent and Dependent Events)

View Set